Vous êtes sur la page 1sur 82

QUESTION 1 Fred Farmer participates in a federal program which pays him to let certain parts of his farm lie

fallow. The agency that administers the federal program failed to act when Farmer requested a change in the way his payments are calculated. Oddly enough, Farmer's request would actually correct a problem that has existed because the agency has failed to follow its own regulations. The agency claims its regulations are merely guidelines and it is not required to follow them. The regulations, on their face, are mandatory and mirror the controlling statute. Both the regulations and the statute provide that payments to farmers shall be calculated in the manner that Farmer has requested. Farmer demanded, in writing, that the agency make the requested change. The agency failed to respond to Farmers request. Farmer then threatened to take the matter to court. The agency countered by saying it has not made a final decision and therefore, court action is unavailable. Farmer has gone through all available levels of agency review including a request for reconsideration, without success. At no time during this process has the agency given any substantial justification for its failure or refusal to act.

QUESTION: Discuss the remedies available to Farmer and the standards that he will have to meet to obtain relief from a court.

2/08

DISCUSSION FOR QUESTION 1 This question raises issues of administrative law and remedies for lack of action, or improper or unlawful administrative action, pursuant to the Administrative Procedure Act, 5 U.S.C. 701 et seq. Agency action made reviewable by statute and final agency action for which there is no other adequate remedy in a court are subject to judicial review. A preliminary, procedural, or intermediate agency action or ruling not directly reviewable is subject to review on the review of the final agency action. Except as otherwise expressly required by statute, agency action otherwise final is final for the purposes of this section whether or not there has been presented or determined an application for a declaratory order, for any form of reconsideration, or, unless the agency otherwise requires by rule and provides that the action meanwhile is inoperative, for an appeal to superior agency authority. 5 U.S.C. 704. A person who seeks judicial review of an agency action must have exhausted the agency's appeal procedure(s). See APA 704. In addition, the prerequisites to judicial review of agency action, in the absence of other statutory provisions, are final agency action and the absence of any other adequate remedy. Klein v. Commissioner of Patents of U.S., 474 F. 2d 821, (C.A. Va..) The person must also have suffered a legal wrong. See 5 U.S.C. 702; and see Duba v. Shuetzle 303 F.2d 570, 574 (8th Cir. 1962). The facts indicate that in this situation, Farmer has utilized the in-house agency appeal procedure. Therefore, exhaustion of administrative remedies is not an issue. Review of an administrative agency decision involving a federal program, if available, will normally be in the federal district courts, which have original jurisdiction of all civil actions under the constitution or laws of the United States. 28 U.S.C. 1331. There is a strong presumption that all agency actions are reviewable under the APA. Woodsmall v. Lyng, 816 F.2d 1241, 1243 (8th Cir. 1987); and see 5 U.S.C. 702. The central purpose for judicial review under the Administrative Procedure Act is to provide "a broad spectrum of judicial review of agency action." Bowen v. Massachusetts, 108 S.CT. 2722, 487 U.S. 879 (1988). With regard to these facts, under the APA 706, the reviewing court may: 1. compel agency action unlawfully withheld or unreasonably delayed; and 2. hold unlawful and set aside agency action, findings, and conclusions found to be a. arbitrary, capricious, an abuse of discretion, or otherwise not in accordance with law; b. contrary to constitutional right, power, privilege, or immunity;

DISCUSSION FOR QUESTION 1 Page Two c. in excess of statutory jurisdiction, authority, or limitations, or short of statutory right; d.. without observance of procedure required by law; e. unsupported by substantial evidence in a case subject to sections 556 and 557 of this title or otherwise reviewed on the record of an agency hearing provided by statute; or f. unwarranted by the facts to the extent that the facts are subject to trial de novo by the reviewing court. The standard of review of agency action by a district court is rather narrow, and while the court reviews the entire record, it must defer to the agency's interpretation of its regulations. Chevron U.S.A. v. Natural Resources Def. Council, Inc., 467 U.S. 873 (1984). Reversal can only occur when the agency action is without a rational basis. Baltimore Gas & Elec. Co. v. National Resources Defense Council, 462 U.S. 87, 105-06 (1963). The reviewing court examines an agency's conclusions of law de novo, but it must uphold the agency's factual findings if they are supported by "substantial evidence." That is defined as "more than a mere scintilla but less than a preponderance." Andrews v. Shalala, 53 F.3d 1035, 1039 (9th Cir. 1995). Administrative agencies are limited in their powers by the congressional acts which grant them authority. Garvey v. Freeman, 397 F. 2d 600, (C.A. 10. Colo. 1968). In order to determine whether an agency acts within the scope of its authority, the court must review the scope of the agency's authority and whether the agency is acting within that range. Olenhouse v. Commodity Credit Corp., 42 F. 3d 1560 (C.A. 10. Kan., 1994). A reviewing court must examine the relevant statutes to determine whether an agency has acted within the scope of its authority. Lodge Tower Condominium Ass'n v. Lodge Properties, Inc., 880 F. Supp. 1370 (D. Colo., 1995). Here, the agency has not acted within its authority nor has it complied with regulatory requirements. If an agency fails to follow its own regulations that is an abuse of discretion. Carter v. Sullivan, 909 F.2d 1201, 1202 (8th Cir. 1990). The agency here contends that its action was not "final agency action." Federal courts have considered questions of finality in many cases, including in the case of Coalition for Sustainable Resources, Inc. v. United States Forest Service, 259 F. 3d 1244, 1249 (10th Cir. 2001). In that case, the court principally considered issues of "ripeness" to determine whether there was final agency action. Although cases of an agency's failure to act are somewhat problematic, the court in Coalition, id., stated that an examination must include not only fitness of the issues for a decision but also hardship of the parties if the court withholds action. "An agency cannot preclude judicial review by casting its decision in the form of inaction rather than in a form of an order denying relief." Id. at 1251. An action may be final when an agency either refuses to act, unreasonably delays, or fails to act before a deadline. Id. In a case where an agency refused to consider a fee application under EAJA, that was a final determination and reviewable by the Court. Lane v. US Dept. of Agriculture, 629 F. Supp. 1290, D.N.D. 1996, affirmed in part and reversed in part, 120 F. 3d 106.

DISCUSSION FOR QUESTION 1 Page Three

"Finality" generally refers to the conclusion of agency activity. Bethlehem Steel Corp. v. E.P.A., 669 F. 2d 903 (C.A. 3rd 1982). 5 USC 706 provides for compelling of agency action which has been unlawfully withheld or unreasonably delayed. That has apparently happened in this case. The agency cannot be allowed to simply refuse or neglect to act, and then contend as a result of such refusal or neglect that final action has not occurred. Under 5 USC 706, if action is unlawfully withheld or unreasonably delayed, the agency has presumably not "finalized" its action. However, that is not an excuse nor is it a justification to deny judicial review and enforcement under the APA. See also, Coalition for Sustainable Resources, supra at 1250, citing Sierra Club v. Yeutter, 911 F. 2d 1405, 1410 (10th Cir. 1990). The agency's failure and refusal to follow its own rules and statutes may constitute a violation of Farmers property and due process rights. When questions of due process are the subject of appeal from an agency final decision, the District Court must conduct a plenary review of the facts and the agency's decision making process. Olenhouse v. Commodity Credit Corp., 42 F. 3d 1560, 1565 (10th Cir. Kan. 1994). The District Court "must find and identify substantial evidence to support the agency's actionagency action must be set aside if it fails to meet statutory, procedural or constitutional requirements or if it was 'arbitrary, capricious, an abuse of discretion, or otherwise not in accordance with law.'" Olenhouse, supra, at page 1565, 1574, citing Citizens to Preserve Overton Park v. Volpe, 401 U.S. 402, 91 S.Ct. 814, 28, L.Ed.2d 136 (1971). Agency action will also be set aside if the administrative process employed violated "basic concepts of fair play." Olenhouse, supra, at 1583. The same theory applies where the agency has failed or refused to act, especially where the agency "failed to take a discrete agency action that it is required to take. Norton v. Southern Utah Wilderness Alliance, 124 S.Ct. 2373 (2004) at pg 2380. As applied to this case, Farmer is likely to be successful in obtaining judicial review of the agency's action, or more properly, the agency's failure to act. The agency appears to have clearly violated or ignored its own regulations and has taken action which is contrary to those regulations. Not only is this potentially unlawful, or arbitrary and capricious, but it may also be in excess of statutory authority or limitations, and without observance of procedure that is required by law. (5 U.S.C. 706). Even though the agency's interpretation of its regulations is entitled to deference, this is probably a case of a clear error of judgment or an abuse of discretion by failure to follow its own regulations. Citizens to Preserve Overton Park, Inc., supra, and Carter v. Sullivan, supra. Farmer is therefore, likely entitled to judicial review of the administrative action and relief which either compels agency action or sets aside unlawful action.

2/08

COLORADO SUPREME COURT Board of Law Examiners

FEBRUARY 2008 BAR EXAM Regrade

ESSAY Q1
ISSUE

SEAT
POINTS AWARDED

1. 2. 3. 4.

There is a general rule requiring the exhaustion of administrative remedies before a court will consider judicial review of an administrative agency decision. In order to bring an action for judicial review, Fred must have standing. For standing, Fred must be within the zone of interest ("person injured or affected"). Prerequisites for judicial review of agency action are: 4a. "final" agency action; 4b. 4c. the absence of any other adequate remedy ("redressible"); and person must have suffered a legal wrong ("harmed").

1. 2. 3.

4a. 4b. 4c. 5. 6.

5. 6. 7.

In this case, Fred has already utilized the agency appeal procedures and therefore exhausted administrative remedies. Review of federal administrative agency decisions would be in federal district court. A reviewing court may compel agency action unlawfully withheld or set aside agency action found to be: 7a. 7b. 7c. 7d. 7e. 7f. arbitrary, capricious or abuse of discretion; contrary to a constitutional right, power, privilege or immunity; in excess of statutory jurisdiction or authority; without observance of procedure required by law; unsupported by substantial evidence in the case or hearing; or unwarranted by facts in an applicable de novo hearing.

7a. 7b. 7c. 7d. 7e. 7f. 8. 9. 10. 11. 12.

8. 9. 10. 11. 12.

Under the standard of review of agency action by the court, it must defer to the agency's interpretation of its regulation. A review court must uphold an agency's factual findings if supported by substantial evidence. In this case, Fred has a strong argument that the agency was not acting within its authority prescribed by the federal statutes and the mandatory regulations. In this case, Fred could claim that failure by the agency to follow its own regulations. Even though the agency is claiming that it has not taken "final" action, an action may be final when an agency refuses to act, unreasonably delays, or fails to act before a deadline ("futility"). Fred may bring a claim for violation of his due process rights.

13.

13.

9474050021

BLE Gradesheet v2.1

page 1 of 1

QUESTION 2 Father wrote to his adult Son, I want you to have my property Twelve Oaks as a wedding present, but I would need $50,000 from you to pay off the mortgage on the property. Son replied in writing, I will pay you $50,000 for Twelve Oaks on March 1, provided I can get a loan from the bank before that date. Father replied by mail, It's a deal. Both Father and Son knew that Twelve Oaks was reasonably worth $100,000. Although he tried, Son could not obtain a loan from the bank. Instead, his mother-in-law lent him $50,000. Son then paid the $50,000 to Father on March 2 and explained that he was out-of-town on business on March 1, and returned too late to make payment on that date. Father accepted the money and discharged the mortgage. Later, however, having learned that Son obtained the $50,000 from his mother-in-law and not from the bank, Father changed his mind about the wedding present and the sale and refused to deed Twelve Oaks to Son. Father gave as his reasons: (1) that there was no consideration to support the deal; (2) that the condition of obtaining a loan from the bank had not occurred; and (3) that Son was late in paying the $50,000.

QUESTION: Discuss the validity of Fathers reasons for not delivering the deed to Twelve Oaks to Son.

2/08

DISCUSSION FOR QUESTION 2 The exchange of writings by Father and Son would effect an enforceable contract if there is consideration to support Father's promise to deliver the deed. The agreement is in writing and all the essential terms of a land contract are present parties, description of the property, price, and time of performance. Consideration consists of an act, forbearance, or return promise, bargained for and given in exchange for the promise. Restatement (Second) of Contracts, 71. On these facts, the only thing that could be consideration is Son's promise to pay $50,000. The courts are not concerned about the adequacy of the consideration or that what is bargained for is the equivalent of what was promised. Id. at 79(b). If it is bargained for it is irrelevant that Son is promising to pay only one-half the value of Twelve Acres. In this case, Father has two motives for deeding Twelve Oaks to Son to make a wedding gift (which cannot serve as consideration), and to receive $50,000 from Son. Even where both parties know that a transaction is in part a bargain and in part a gift, the element of bargain (here Son's paying $50,000) may nevertheless furnish consideration for the entire transaction. Id. at 71, comment c. It is clear that Father is bargaining for the $50,000 so that he can pay the mortgage debt, and, therefore, there is bargained-for exchange to support his promise to deliver the deed. See Id. at 71, Illus. 6. There is no doubt that obtaining the loan from the bank was a condition to Son's duty to pay $50,000. The question is whether it was also a condition to Father's duty to deliver the deed. Since the origin of the money should make little or no difference to Father, in this kind of situation the courts will interpret the condition as applying only to Son's duty to pay. Farnsworth, Contracts, 3rd Ed., 8.4. Son has waived that condition, and so his duty to pay arose even though the condition was not met. Id. at 8.4. Since the bank loan was not a condition to Father's duty to deliver the deed, Father's duty to deliver the deed arose when he accepted Son's $50,000 payment. Unless it is clear that payment on time is essential to protect the promisor, courts are reluctant to conclude that late payment excuses the promisor from performing his promise. Absent other indications in the contract to the contrary, time of payment is not interpreted as a condition in a land contract. Id. at 8.18. Although Father may have an action for any damages he may have suffered because of the late payment, he cannot refuse to perform his promise to deliver the deed because of the one-day delay. Even if payment on time was a condition to Father's duty to deliver the deed, Father waived that condition when he accepted the payment, and, therefore, his duty to deliver the deed arose whether or not the condition was payment on time.

2/08

COLORADO SUPREME COURT Board of Law Examiners

FEBRUARY 2008 BAR EXAM Regrade

ESSAY Q2
ISSUE

SEAT
POINTS AWARDED

1.

A valid Contract between Father and Son exists, as all of the elements are present. 1a. 1b. 1c. Offer Acceptance Consideration 1a. 1b. 1c. 2. 3. 4. 5. 6. 7. 8.

2. 3. 4. 5. 6. 7. 8.

Since this is a Contract involving land, it must be in writing to comply with the Statute of Frauds. Son has given valid consideration for the Contract by his promise to pay $50,000. The adequacy of consideration to Father is not a concern under the law, only the existence of consideration. The obligation of obtaining the loan from the Bank is not a condition of Father's duty to deliver the Deed. Unless the Contract is clear that the date of payment is essential to protect the parties, late payment does not excuse performance. Father cannot refuse to deliver the Deed because payment is late; he can only recover for damages he can prove by the delay. Father closed on the contract and waived any condition that payment must be made by March 1 when he accepted payment on March 2.

2543053811

BLE Gradesheet v2.1

page 1 of 1

DISCUSSION FOR QUESTION 3 The question raises several issues in tort. One of the issues raised is the distinction between the torts of battery and assault. Unlike battery, assault requires apprehension but no contact. Therefore, Dave is likely liable to Ref for assault but not for battery because Ref saw the snowball coming but moved just in time to avoid being struck. Dave is likely liable to Star for battery but not for assault because Star did not see the snowball coming but was actually struck by the snowball. Another key issue in tort that is raised is the difference between torts requiring a showing of intent versus those requiring mere negligence. Given that distinction, Dave is also likely liable to Coach, at least, in negligence. I. ASSAULT The elements of the tort of assault are: (1) an act; (2) intent; (3) causation; (4) apprehension of imminent harmful or offensive contact; and, (5) lack of consent by the plaintiff. CJI-Civ. 20:1; Restatement (Third) of Torts 5, Restatement (Second) of Torts 21, 24, and 33. The elements of assault in actions by Ref and Star would likely be resolved as follows: Element Act? Intent? Causation? Apprehension? Lack of Consent? Ref v.Dave yes yes yes yes yes Star v.Dave yes yes yes no yes

To prove intent, Dave need only be shown to have acted with either the intent to cause the type of harm suffered or the intent to do the act that is substantially certain to cause that type of harm. Restatement (Third) of Torts 1. Although Dave had no intent to harm Star, his intent to harm Ref can be transferred to Star. Id.at 33, CJI-Civ. 20:8 (It is not necessary that the defendant intended to make physical contact specifically with the plaintiff. Intent exists even if the defendant originally intended to make physical contact with someone else.). Likewise, he may not have intended to harm anyone in particular. Dave will argue that Ref suffered no true apprehension because apprehension must be somewhat significant. The plaintiff must have been in fear of imminent physical harm, not merely have suffered a fright. Restatement (Second) of Torts 24 n.b. Still, Ref might argue that he suffered significant apprehension, the amount of which goes only to the potential for nominal damages. CJI-Civ. 20:4 n. 7.

DISCUSSION FOR QUESTION 3 Page Two Dave might also argue that, by consenting to take the field during an admittedly violent activity (football), the various plaintiffs had somehow consented to a risk of injury while on the field; he might try to argue that, in fact, the threat of harm posed by his action (throwing a snowball) is generally less than any posed by football. However, his argument would not be well founded, as ones consent only bars torts that pose the same or substantially the same risk posed by the tort to which the plaintiff has consented. Restatement (Second) of Torts 892(A), CJI-Civ. 20:11. While the various defendants may have consented to the athletic risks associated with a game of football, there is no evidence that any consented to the risk of projectiles thrown by spectators.

II. BATTERY The elements of the tort of battery are: (1) an act; (2) intent; (3) causation; (4) harmful or offensive bodily contact; and, (5) lack of consent by the plaintiff. CJI-Civ. 20:5 and 6; Restatement (Third) of Torts 5, and Restatement (Second) of Torts 13-20. The elements of battery in actions by Ref and Star would likely be resolved as follows: Element Act? Intent? Causation? Contact? Lack of Consent? Ref v. Dave yes yes yes no yes Star v. Dave yes yes yes yes yes

III. INTENTIONAL INFLICTION OF EMOTIONAL DISTRESS Any of the potential plaintiffs might consider asserting a claim for the intentional infliction of emotional distress. The elements of the tort of intentional infliction of emotional distress, a/k/a IIED, a/k/a outrageous conduct, are: (1) an act (by Dave) of outrageous conduct; (2) intent; (3) causation; and, (4) damages, including at least severe emotional distress. Restatement (Third) of Torts 45, CJI-Civ. 23:1. To constitute outrageous conduct, the conduct must be so extreme that a reasonable person would exclaim, Thats outrageous. Restatement (Third) of Torts 45. In other words, a reasonable person would view it as exceeding all possible bounds of decency and utterly intolerable in a civilized community, quoting CJI-Civ. 23:2. Dave will argue that the simple act of throwing a snowball, as bad and irregular as this was, is still not that far outside of normal social function. Additionally, he can argue that a single incident, such as this, is also less likely to constitute outrageous conduct. Id., n. 5.

DISCUSSION FOR QUESTION 3 Page Three Dave also will argue that damages in such a claim must at a minimum include severe emotional distress. Restatement (Third) of Torts 45. Severe emotional distress consists of highly unpleasant mental reactions, such as (nervous shock, fright, horror, grief, shame, humiliation, embarrassment, anger, chagrin, disappointment, or worry) and is so extreme that no person of ordinary sensibilities could be expected to tolerate and endure it. The duration and intensity of emotional distress are factors to be considered in determining its severity. CJI-Civ. 23:4. Some jurisdictions also require physical injury as a component of the plaintiffs damages. Restatement (Third) of Torts 45, Restatement (Second) of Torts 46(k). According to those definitions, Dave will argue that none of the plaintiffs suffered severe emotional distress.

IV. NEGLIGENCE All three Ref, Star and Coach may also have a claim of negligence against Dave. If a tortfeasors mental state is insufficient to constitute intent, it may still be sufficient to constitute negligence. Restatement (Third) of Torts 1(d). The elements of a negligence claim are: (1) a duty owed by Dave; (2) a breach of that duty through the commission of negligent act; (3) causation; and, (4) damages. CJI-Civ. 9:1. Dave owed a duty to exercise reasonable care. Restatement (Third) of Torts 7, CJI-Civ. 9:6 and 8. He may argue that he did not even consider Coach, but Coach will argue it was reasonably foreseeable that he, and anyone else standing on the field, could have been injured by Daves act; therefore, Dave would owe them all a duty not to be negligent. Dave also may argue he did not cause Coachs injury. To prove causation, Coach must show that Daves act of throwing the snowball was both the actual (but for) and proximate (legal) cause of his injuries. Restatement (Third) of Torts 26, 27, and 29, CJI-Civ. 9:18 and 21. He will argue it was reasonably foreseeable that someone, at whom a snowball is thrown, would move as both Ref and Star did and, further, that someone could be hurt, as Coach was, especially because the players, like Star, are large, powerful, and effectively armored. In other words, Coach could argue Ref and Stars actions were foreseeable reaction forces for which Dave is liable, not an intervening force. Restatement (Third) of Torts 34.

V. NEGLIGENT INFLICTION OF EMOTIONAL DISTRESS The elements of the tort of negligent infliction of emotional distress are: (1) an act of negligence; (2) that created an unreasonable risk of physical harm to the plaintiff; (3) that caused the plaintiff to be in fear of his safety (not simply a momentary fright); (4) causation; and, (5) damages. Restatement (Third) of Torts 46 and 47, CJI-Civ. 9:2. This tort only applies if the plaintiff was himself put in an unreasonable risk of physical harm, a/k/a, within the target zone or zone of danger. (Although some exceptions exist to permit claims by family members, such as parents of children, no such exception is raised by these facts.) Restatement (Third) of Torts 46 and 47, Restatement (Second) of Torts 26.

DISCUSSION FOR QUESTION 3 Page Four VI. DAMAGES Dave will argue that Ref and Star had little to no damage. Even then though, they might claim nominal damages and the value of any mental anguish they suffered. Dave also may argue that Coachs damages were not caused by his act of throwing the snowball but more so, or at least in part, by Stars movement to jump into Coach. Restatement (Third) of Torts 34, CJICiv. 9:20. The argument can be viewed as one of damages (e.g. comparative fault) and/or causation (intervening force). Restatement (Third) of Torts 34 n. c-d. As explained, Coach will argue, though, that Dave is liable under both theories since Stars action was the sort Dave should reasonably have foreseen (reaction force). None would recover attorney fees.

2/08

COLORADO SUPREME COURT Board of Law Examiners

FEBRUARY 2008 BAR EXAM Regrade

ESSAY Q3
ISSUE

SEAT
POINTS AWARDED

1.

Dave may be liable for assault. 1a. The elements of assault include intent, apprehension and lack of consent.

1. 1a. 2. 3. 3a. 4. 4a. 5. 6. 7. 7a. 8. 8a. 9. 10. 11.

2. 3.

Apprehension must be reasonable. Dave may be liable for battery. 3a. The elements of battery include intent, contact and lack of consent.

4.

Dave may be liable for intentional infliction of emotional distress (outrageous conduct). 4a. The elements of IIED (OC) include intent, outrageous conduct and severe emotional distress.

5. 6. 7.

Outrageous conduct is conduct so outrageous that a reasonable person would say "outrageous!" (or, say it was beyond all possible bounds of decency). Intent can be transferred (from Dave's intent v. Ref to Star and perhaps even Coach). Dave may be liable for negligence. 7a. The elements of negligence include duty of reasonable care and breach (negligent act).

8.

Dave may be liable for negligent infliction of emotional distress. 8a. The elements of NIED include an unreasonable risk of physical harm to plaintiff and actual fear (not just "momentary fright").

9. 10. 11.

NIED does not apply unless the plaintiff was put in an unreasonable risk of physical harm ("target zone" or "danger zone"). Causation requires both actual ("but for") cause and proximate ("legal") cause. Whether the mousetrap-like series of events is viewed as an issue of duty or causation/damages, the plaintiffs' damages must have been"reasonably foreseeable."

1729055065

BLE Gradesheet v2.1

page 1 of 1

QUESTION 4 Last month, on a rural tract of land located in the State of Bliss, the Ku Klux Klan (KKK) held a membership rally. A secretly made film of the rally shows twelve hooded figures gathered around a large wooden cross, carrying firearms, and ultimately burning the cross. During the rally, speakers made derogatory references about ethnic and religious groups. One speaker, Jones, stated Were not a vengeful organization, but if our President, our Congress, our Supreme Court, continue to suppress the white, Caucasian race, its possible that there might have to be some revenge taken. When the film was made public, Jones was arrested and charged under two Bliss statutes. One statute, Blisss Syndicalism Statute, makes it a crime to advocate the duty, necessity or propriety of crime, sabotage, violence, or unlawful methods of terrorism as a means of accomplishing industrial or political reform, and also prohibits people from voluntarily assembling with any society, group or assemblage of persons formed to teach or advocate the doctrines of criminal syndicalism. The second statute, Blisss Cross Burning Statute, makes it unlawful for any person or persons, with the intent of intimidating any person or group of persons, to burn, or cause to be burned, a cross on the property of another, a highway, or other public place. Any person who shall violate any provision of this section shall be guilty of a Class 6 felony. The Cross Burning Statute goes on to state that: Any such burning of a cross shall be prima facie evidence of an intent to intimidate a person or group of persons.

QUESTION: Discuss whether in light of protections offered under the First Amendment to the United States Constitution, Jones will be convicted under the Bliss statutes.

2/08

DISCUSSION FOR QUESTION 4

The Federal Rules of Evidence (F.R.E.) will determine the admissibility of each item of evidence. 1. Defendants Statement

Officers testimony that Defendant told him at the accident scene that she entered the intersection on a green light is hearsay. Hearsay is an out-of-court statement offered for the truth of the matter asserted therein. F.R.E. 802. Here, the statement is being offered for its truth to prove that the light was green, thus eliminating Defendants liability. Hearsay is inadmissible unless the particular hearsay satisfies one of the exceptions to the hearsay prohibition. Here, the following arguments may be made. First, Defendant will argue that her statement constitutes a present sense impression. This exception applies to hearsay that describes or explains an event or condition and is made while the declarant was perceiving the event or condition, or immediately thereafter. F.R.E. 803(1). Defendant is the declarant, and although her statement does describe the event, she was not observing the event (her automobile entering the intersection) while she made the statement. Thus, she must argue that her statement was made immediately after the event. This argument will likely fail. Although courts have not identified a rigid time frame that constitutes immediately thereafter, see United States v. Blakey, 607 F.2d 779 (7th Cir. 1979) (23 minutes was immediately thereafter); Hilyer v. Howat Concrete, 578 F.2d 422, 426 n.7 (D.C. Cir. 1978) (15 minutes or more too long), the better answer in this case is that Defendant statement came too long after the event. Defendant was intimately involved in the accident, and the passage of time coupled with the police investigation gave Defendant motivation to fabricate. Next, Defendant may argue that her statement constitutes a party admission. Party admissions are not hearsay. Fischer v. Forestwood Co., Inc., 525 F.3d 972 (10th Cir. 2008). F.R.E. 801(d)(2). Although Defendants statement was made by a party to the litigation, it was not offered into evidence by Plaintiff, her party-opponent. The statement thus does not constitute an admission by a party opponent under F.R.E. 801(d)(2). Defendant may also argue that her statement is admissible for a non-hearsay purpose, as a prior consistent statement. F.R.E. 801(d)(1)(B). For this theory to be applicable, Defendant would have to testify and be subject to cross examination. Her testimony would have to be impeached as the product of a motivation to fabricate, and her statement to Officer would have to precede her motivation. There is nothing in the facts to indicate a motive to fabricate. This could only be the fact if, for example, Defendant made the statement and then an intervening motive to fabricate arose - such as Defendant being paid to change her testimony. Tome v. United States, 513 U.S. 150 (1995); United States v. Gomez, 191 Fed.Appx. 413 (6th Cir.2006). As no facts indicating such an intervening event or changed testimony are present here, the better answer is that this argument should fail

2.

Defendants Drivers License

Any evidence that tends to make a material issue more or less true is relevant. F.R.E. 402. Officers testimony that Defendants license reveals a driving restriction, and that she was without her corrective lenses, is relevant because her need for corrective lenses when driving, and her failure to wear them, would help establish Plaintiffs claim of negligence. Defendant would object to this evidence on the grounds of the best evidence rule. This rule excludes testimony about the contents of a writing, photograph or recording, and instead requires the proponent to produce the original, a duplicate, or an excuse for its non-production. F.R.E. 1002. The court would probably sustain the objection. Although Officer may testify to his observation that Defendant was not wearing corrective lenses, he may not testify to the contents of the writing on the license; he must produce the license, a duplicate, or an excuse for its non-production. 3. Police Report

Defendant offers Officers police report, which contains a description of the accident scene and records a statement from Plaintiff stating that he had been heading home from a bar when the accident occurred. Officer testified that he prepares such reports routinely at accident scenes, and is always careful to record the facts and statements accurately. Police reports are generally inadmissible in criminal cases. F.R.E. 803(8); United States v. Oates, 560 F.2d 45 (2d Cir. 1977). This is a civil case, however, so this prohibition will not apply. See Truesdale v. Klich, , 2006 WL 1460043 (N.D.Ill.,2006) (police reports, which recorded first-hand observations of testifying officers, were admissible in civil trial under the public records and reports exception to the hearsay rule); Bolduc v. United States, 265 F.Supp.2d 153 (D.Mass.,2003 (same). The evidence contains hearsay within hearsay. Officers police report, a hearsay document, contains Plaintiffs hearsay statement. Hearsay within hearsay is admissible as long as each embedded hearsay statement falls within one or more hearsay exceptions. F.R.E. 805. Plaintiffs statement constitutes a party admission under F.R.E. 801(d)(2). It is a statement by a party offered into evidence by a party-opponent, here Defendant. It may also be used to impeach Plaintiffs credibility if Plaintiff offers contradictory evidence at trial. In such an event, the testimony is only narrowly admissible for impeachment as to credibility. One could also argue that Plaintiffs statement fits within the statement against interest hearsay exception in F.R.E. 804(b)(3). This exception applies to statements that are so far contrary to the declarants interests that no reasonable person would have made them unless they were true. F.R.E. 804(b)(3). This argument would likely be unsuccessful because this exception is available only where the declarant (here, Plaintiff) is unavailable as a witness. No facts in the problem suggest that Plaintiff, a party, is not available to testify. Officers police report is a business record. It is a record made contemporaneously with the events Officer was describing (the accident scene and the statements by the participants) and

was made as a regular part of Officers business. F.R.E. 803(6). Officers testimony lays the requisite foundation for this exception to apply. Defendant may thus offer the report; it is admissible because it is a business record and the hearsay statement contained therein is a party admission. See Truesdale, supra; Bolduc, supra. 4. Plaintiffs Prior Conviction

Prior convictions may be used to impeach the testimony of a witness. F.R.E. 609. When the nature of the crime that underlies the prior conviction involves dishonesty or a false statement, that conviction is admissible to impeach even if it is a misdemeanor and without any consideration of the potential unfair prejudice from the impeachment under F.R.E. 403. F.R.E. 609(a)(2). Fraud is a crime involving dishonesty or a false statement. The conviction occurred within the last ten years, and thus the prohibition on the use of convictions more than ten years old does not apply. F.R.E. 609(b).

7/08

COLORADO SUPREME COURT Board of Law Examiners

FEBRUARY 2008 BAR EXAM Regrade

ESSAY Q4
ISSUE

SEAT
POINTS AWARDED

1. 2.

First Amendment protects freedom of speech and expression. First Amendment applies to states via the due process clause of the 14th Amendment. STATUTE ONE: SYNDICATE STATUTE

1. 2.

3. 4. 5. 6. 7. 8. 9. 10.

Jones' oral speech constitutes "political expressions" within First Amendment. Even oral speech that advocates violence or illegal action is protected. Content based & viewpoint based prohibition on free speech generally not allowed. Statute can forbid advocacy of use of force/violation of law where speech is directed to inciting imminent lawless "fighting words" action and is likely to produce such. Jones statement, 'possible there might have to be some revenge taken' not likely to produce imminent lawless action or incite such action. Statute forbids assembly with others to advocate actions protected by First Amendment. Bliss Syndicate Statute is unconstitutional/not valid. Jones will not likely be convicted under the Bliss Syndicate Statute. STATUTE TWO: CROSS BURNING

3. 4. 5. 6. 7. 8. 9. 10.

11. 12. 13. 14. 15. 16.

Symbolic expression, such as cross burning, is protected by the First Amendment. State can prohibit cross burning if combined with intent to intimidate true threat. The prima facie evidence provision of the Bliss Statute doesn't allow analysis of 'intent to intimidate.' Bliss cross burning statute is unconstitutional/not valid. No intent to intimidate: Rally where cross burned held on private property of group member (or) not directed at individual or group (no intent to intimidate) Jones will not likely be convicted under the cross burning statute.

11. 12. 13. 14. 15. 16.

0119056675

BLE Gradesheet v2.1

page 1 of 1

QUESTION 5 On May 1, Amy and Bill entered into an oral agreement to open a dance studio called Kickers. Kickers leased space from Dubliner, agreeing to pay Dubliner 15% of the gross fees Kickers collected from its students for the right to use the leased space. Dubliner had no involvement in the management or operation of Kickers. The lease required a deposit of $500 which Amy paid. Amy and Bill both taught classes, and Bill handled the bookkeeping. They agreed to split the profits equally. On May 15, Amy signed a contract with a sign fabricator to make a store-front sign for Kickers. The contract required Kickers to pay $4,000 for the design and fabrication of the sign and a monthly fee of $200 for a pole on which to display the sign. Unbeknownst to Amy, on June 1, Bill started giving some of the more competitive dancers private classes in his basement, keeping the money he earned from those classes. He told the students Kickers was using his basement as an annex. On July 1, a student tripped on loose carpeting in Bills basement and was injured. Business was booming at Kickers, so on July 15, Amy and Bill hired another dance instructor, Maureen. Soon thereafter, they sold Maureen a 10% ownership interest in Kickers for $10,000 and deposited the money in Kickers business account. Maureen agreed to share equally in the profits of Kickers. On September 1, Amy and Maureen discovered Bills side business when the injured student sued Amy, Bill, Maureen, Kickers and Dubliner. They also discovered that Bill had failed to pay the sign fabricator.

QUESTIONS: Discuss: 1. 2. the nature of the relationships between Amy, Bill, Maureen, and Dubliner; which of the defendants can be held liable for the students injuries and the debt to the sign fabricator; the extent of each partys liability (if any); and, what claims Amy and Maureen can assert against Bill.

3. 4.

2/08

DISCUSSION FOR QUESTION 5 I. Relationships Between the Parties A. Amy and Bill are General Partners The Revised Uniform Partnership Act (RUPA) defines a partnership as an association of two or more persons to carry on as co-owners of a business for profit. The parties agreement need not be in writing to form a valid partnership. Community Capital Bank v. Fischer & Yanowitz, ___ N.Y.S.2d ___ (N.Y.A.D. 2 Dept.,2008); Montgomery County v. Wildwood Medical Center, L.L.C., 176 Md.App. 731, 934 A.2d 484 (2007)(dissenting opinion); Cooley Inv. Co. v. Jones, 780 P.2d 29 (Colo. App. 1989); 7-60-106(1), C.R.S. 2007. A partnership may be formed by the conduct of the parties. Qubain v. Granberry, 181 N.C.App. 149, 639 S.E.2d 454 (2007). Partners can contribute property, money or services to the partnership and their contributions need not be equal. In re Estate of Goldstein, 293 Ill.App.3d 700, 688 N.E.2d 684 (1997); Kennedy v. Miller, 221 Ill.App.3d 513, 582 N.E.2d 200 (1991). The sharing of profits is prima facie evidence of a partnership. Mardanlou v. Ghaffarian, 135 P.3d 904 (Utah App. 2006); Yoder v. Hooper, 695 P.2d 1182 (Colo. App.1984); 7-64202(3)(c), C.R.S. 2006. Here, Amy and Bills verbal agreement and conduct establish a partnership. They agreed to share profits equally, and both contributed to the partnership. The fact that Amy contributed money and services, while Bill contributed only services, is immaterial. Kickers was a general partnership when it was formed and Amy and Bill are general partners. B. Dubliner is a Landlord, not a Partner Amy, Bill, Maureen, and Kickers have a landlord-tenant relationship with Dubliner. Dubliner has no involvement in the management of Kickers. The fact that Dubliner receives a share of profits as rent does not make Dubliner a partner. See Ehrhardt v. Abbate, 2002 WL 1265569 (Ohio App. 8 Dist.); 7-64-202(3)(c)(III), C.R.S. 2007; 14-8-7(4)(c), Ga. Stat. Ann. 2007. C. Maureen is a Partner Maureen started out as an employee or independent contractor of Kickers, but became a partner when she bought an ownership interest. II. Potential Liabilities of the Parties A. Amy Partners are jointly liable for the debts and obligations of the partnership, whether the obligations are in contract or tort. Ederer v. Gursky, ___ N.E.2d ___ (2007 WL 4438937); Shar's Cars, L.L.C. v. Elder, 97 P.3d 724 (Utah App.,2004); Zimmerman v. Dan Kamphausen Co., 971 P.2d 236 (Colo.App. 1998).

DISCUSSION FOR QUESTION 5 Page Two

A partnership is liable for injury caused to a person as a result of a wrongful act or omission, or other actionable conduct, of a partner acting in the ordinary course of business of the partnership. Partners are liable for any torts committed by a partner in the ordinary course of partnership business or with authority of the partnership. Ederer v. Gursky, supra; Gildon v. Simon Property Group, Inc., 158 Wash.2d 483, 145 P.3d 1196 (2006). An act of a partner that is apparently carried out in the ordinary course of partnership business binds the partnership and other partners, unless the partner has no authority to act for the partnership in the particular matter and the person with whom the partner was dealing had notice that the partner lacked authority. See Ederer v. Gursky, supra; see also 7-64-301(1)(a), C.R.S. 2007. Because Bill led the student to believe he was teaching classes in his basement as an extension of Kickers, the student was not on notice that Bill did not have authority to do so. Accordingly, Amy is liable for the students injuries. Amy is also liable for the full amount of the partnership debt to the sign fabricator.

B. Bill As a partner Bill is liable to the sign fabricator because the contract was made by a partner in the scope of the partnership business and was authorized by the partners. (RUPA 305306). Bill is liable as a partner for the injury to the student and is separately liable for the students injuries, since he was the tortfeasor (the fall was caused by Bills negligent maintenance of his carpet).

C. Maureen A person admitted as a partner into an existing partnership is not personally liable for any partnership obligations incurred before the persons admission as a partner. Trizechahn Gateway LLC v. Titus, 930 A.2d 524 (Pa.Super. 2007); 7-62-303, 7-64-306(2) and (4), C.R.S. 2007. Maureen is personally liable only for debts incurred after she became a partner. However, the $10,000 she contributed as capital to the partnership is at risk for satisfying existing partnership obligations. R.U.P.A. 306(b). Thus, she is not liable to the sign fabricator for the sign itself, but can be held liable for the unpaid rent for the sign pole which was incurred after she became a partner. Since the students injury occurred prior to the date of her becoming a partner, Maureen is not liable for the students injuries.

DISCUSSION FOR QUESTION 5 Page Three

D. Kickers A partnership can sue or be sued. Pennsy Corp. v. Pinter, 17 Misc.3d 1116(A) (2007 WL 3037559) (N.Y.Sup. 2007); Gravel Resources of Arizona v. Hills, 217 Ariz. 33, 170 P.3d 282 (Ariz.App. Div. 1, 2007); People ex rel. Totten v. Colonia Chiques, 156 Cal.App.4th 31, 67 Cal.Rptr.3d 70 (Cal.App. 2 Dist.,2007); 7-64-307(1), C.R.S. 2007. As pertinent here, the partnerships liability is the same as the liability of the general partners. See 7-64-301, 7-64305, 7-64-307, C.R.S. 2007. Accordingly, the partnership is liable for the students injuries and the debt to the sign fabricator. All assets of a partnership, including capital accounts, are subject to the claims of creditors. E. Dubliner Because it is not a partner, Dubliner has no liability to either the injured student or the sign fabricator.

III.

Claims Amy and Maureen Can Assert Against Bill

Partners owe a fiduciary duty of loyalty and due care to the partnership and each other and they must discharge their fiduciary duties in a manner consistent with the obligation of good faith and fair dealing. See J & J Celcom v. AT & T Wireless Services Inc., 169 P.3d 823 (Wash. 2007); 7-64-404, C.R.S. 2007. Partners may not compete with the partnerships business, and must disclose to the partnership any benefits or profits they receive without consent of the other partners from any transaction connected with the partnership business. Jarl Investments, L.P. v. Fleck, ___ A.2d (2007 WL 4180969) (Pa.Super. 2007); Yoder v. Hooper, supra; Tucker v. Ellbogen, 793 P.2d 592 (Colo. App. 1989); UPA 404; 7-64-404(1)(c), C.R.S. 2007. Every partner has a right to an accounting as to partnership affairs. Cadwalader, Wickersham & Taft v. Beasley, 728 So.2d 253 (Fla.App. 4 Dist. 1998); Braden v. Strong, (2006 WL 369274) (Tenn.Ct.App. 2006); 7-64-403 and 7-64-404, C.R.S. 2006. The right to an accounting may be enforced by constructive trust for profits which have been wrongfully withheld from the partnership. Bill conducted private classes in the name of the partnership, but kept the profits for himself. Amy and Maureen may demand an accounting and sue Bill for breach of his fiduciary duties. 2/08

COLORADO SUPREME COURT Board of Law Examiners

FEBRUARY 2008 BAR EXAM Regrade

ESSAY Q5
ISSUE

SEAT
POINTS AWARDED

1. 2. 3. 4. 5. 6. 7. 8. 9. 10. 11.

A partnership is an association of two or more persons to carry on as co-owners of a business for profit. The agreement need not be in writing to form a partnership. May be formed by words or express conduct of parties. Although Dubliner receives a share of the profit, its relationship is as landlord and not a partner. Maureen became a partner when she bought an ownership interest. Partners are liable for the debts and obligations of the partnership. A partnership is liable for injuries or claims arising out of the actions of the partners in the ordinary course of business of the partnership. Partners are liable for torts of a partner committed in the ordinary course of partnership business. A person admitted as a partner into an existing partnership is not personally liable for partnership obligations incurred before the admission as a partner. Partners are fiduciaries for the partnership and each other they owe a duty of loyalty to the partnership and to each other. Acts of a partner apparently carried out in the ordinary course of business binds the partnership.

1. 2. 3. 4. 5. 6. 7. 8. 9. 10.

Partners must disclose to the partnership any benefits or profits they receive in the ordinary 11. course of the partnership business and an accounting may be demanded for profits wrongfully withheld. Amy is liable for the student's injuries and for the debt to the sign fabricator. Bill is liable for the student's injuries and for the debt to the sign fabricator. Maureen's $10,000 contribution to the partnership may be used to satisfy partnership obligations. Maureen is not liable for the cost of the sign or the injury to the student because both obligations occurred prior to the time she became a partner. Maureen is liable for unpaid sign rent incurred after she became a partner. Kickers is liable for the student's injuries and for all of the sign company debt. Partners may sue Bill for his breach of fiduciary duties. 12. 13. 14. 15. 16. 17. 18.

12. 13. 14. 15. 16. 17. 18.

4302057865

BLE Gradesheet v2.1

page 1 of 1

DISCUSSION FOR QUESTION 6 I. Validity of the Premarital Agreement

To be valid, a premarital agreement must be in writing and signed by both parties, the parties must make full and fair disclosure regarding their assets and liabilities, and the agreement must be entered into voluntarily without duress, fraud, or overreaching. Section 14-2-307(1), C.R.S. 2007. Here, the facts indicate that the agreement was signed, so it was necessarily also written. The facts also indicate that Fred and Martha had independent counsel during the negotiation process, so the examinees can presume that the agreement was voluntary. The examinees should conclude that, assuming the parties made full financial disclosure, the portions of the agreement regarding the division of property are enforceable. See In re Marriage of Ross, 670 P.2d 26 (Colo. App. 1983) However, the Colorado Marital Agreement Act specifically states that a marital agreement may not adversely affect the right of a child to child support. Section 14-2-304(3), C.R.S. 2007; In re Marriage of Ikeler, 161 P.3d 663 (Colo. 2007); In re Marriage of Chalat, 112 P.3d 47 (Colo. 2005). Thus, the parties agreement that they would bear equal financial responsibility for supporting the children and that neither would be required to pay child support is unenforceable. II. Financial Support of William and Charles A. William Generally, a parents child support obligation continues until the child reaches the statutory age of emancipation, which is 19 in Colorado. However, a child who is serving in the military is considered emancipated, even if he or she is under 19 years old. If William returns to the family before age 19, then child support may be owed. 14-10-115(13)(a)(V), C.R.S. 2007. The facts indicate that William is 18, but neither party can be required to pay child support for him during his service in the military. B. Charles The Colorado statute regarding a parents obligation to pay for a childs college education has changed over the years. After 1997, a court cannot order a parent to pay for any college costs unless the parents entered into an agreement after July 1, 1997 that provides otherwise. Sections 14-10-115(13)(a) and (b), C.R.S. 2007. The facts do not indicate that the parties agreement addressed the issue of post-secondary education. Because their divorce was final in 2007, the post-1997 statute applies, and neither parent can be required to contribute to Charles college expenses. However, because he is 18, and has not yet reached the age of emancipation, either parent can be required to pay child support to the other for Charles.

DISCUSSION FOR QUESTION 6 Page Two III. Pending Motion regarding Child Support for the Baby

The issue of paternity may be raised in conjunction with a determination of child support in a dissolution of marriage proceeding, but the procedures of the Uniform Parenting Act (UPA), 19-4-101, et seq., C.R.S. 2007, must be followed. In re Marriage of De La Cruz, 791 P.2d 1254 (Colo.App. 1990). A man is presumed to be the natural father of a child if he and the child's natural mother are or have been married to each other and the child is born during the marriage . . . [or] within three hundred days after the marriage is terminated. . . . Section 19-4105(1)(a), C.R.S. 2007. A presumption of paternity may be rebutted only by clear and convincing evidence. Section 19-4-105(2)(a), C.R.S. 2007. The burden of proof is on the moving party (in this case, Martha) to establish paternity. C.K.A. v. M.S., 695 P.2d 785 (Colo. App. 1984). Once paternity is established through a court order, the court may enter orders concerning child support. Section 19-4-116(3)(a), C.R.S. 2007. Because Martha is seeking child support, she has the burden of proving Fred is the father of the baby. Fred and Martha were married when the baby was conceived, and the baby was born within 300 days after their divorce became final (the facts indicate that the baby was born 3 months after the divorce). Thus, Fred is the presumptive father. But Martha was married to Paul when the baby was born, so he is also presumed to be the father. The presumption of either as the father may be rebutted. When two or more presumptions arise which conflict with each other, the presumption which on the facts is founded on the weightier considerations of policy and logic controls. Section 19-4-105(2)(a), C.R.S. 2007. However, in weighing competing presumptions, the best interests of the child standard must also be applied N.A.H. v. S.L.S., 9 P.3d 354 (Colo. 2000). Because two presumptions arise here, one or more of the parties will request a genetic test, and the results of the test will determine who the father is. If Fred is the father, he can be required to pay child support for the baby. If the results of the blood test show the probability of Paul as the father, Fred likely will not be required to pay support.

2/08

COLORADO SUPREME COURT Board of Law Examiners

FEBRUARY 2008 BAR EXAM Regrade

ESSAY Q6
ISSUE

SEAT
POINTS AWARDED

1.

To be valid, a premarital agreement must be in writing and signed by both parties, contain full and fair disclosure of each party's assets and financial obligations (liabilities), and the agreement must be entered into voluntarily. The portions of the agreement regarding the division of property are enforceable. Because a marital agreement may not adversely affect a child's right to support, the agreement not to pay child support is unenforceable. Child support obligations continue until the child reaches the age of 19 (emancipation). A child serving in the military is considered emancipated. Because Charles is 18 and has not yet reached the age of emancipation, either parent can be required to pay child support for Charles. However, because there was no agreement to pay for college, the court cannot order either parent to pay for Charles' college expenses. A man is presumed to be the father of a child if the child is born during a marriage. A man is presumed to be the father of a child if the child is born within 300 days of the legal termination of the marriage. Fred may be presumed to be the father because the child was conceived during his marriage to Martha and born within 3 months following the divorce. Paul may also be presumed to be the father because the child was born during his marriage to Martha. It is Martha's burden to proof to establish paternity. If a blood test determines Fred is the father, he can be required to pay child support for the baby.

1.

2. 3. 4. 5. 6. 7. 8. 9. 10. 11. 12. 13.

2. 3. 4. 5. 6. 7. 8. 9. 10. 11. 12. 13.

4158058887

BLE Gradesheet v2.1

page 1 of 1

DISCUSSION FOR QUESTION 7 The issues in this question involve a criminal defendants right to effective assistance of counsel. The Supreme Court has held that the right to counsel is the right to the effective assistance of counsel. McMann v. Richardson, 397 U.S. 759, 771, n. 14 (1970). In Strickland v. Washington, 466 U.S. 668, 691-696 (1984), the United States Supreme Court recognized that the Sixth Amendment to the United States Constitution guarantees the right to effective assistance of counsel. The test for ineffective assistance of counsel requires the defendant to show that counsel provided deficient performance and the deficient performance prejudiced the defendant. Failure to investigate alibi Al met with Defendant at the arraignment and Defendant explained that he had an alibi defense that was home with his mother at the time of the robbery and that he was speeding because he was late for work. Al failed to contact Defendants mother or employer to develop this defense. In assessing the reasonableness of an attorney's investigation, a court would consider not only the quantum of evidence already known to counsel, but also whether the known evidence would lead a reasonable attorney to investigate further. Wiggins v. Smith, 539 U.S. 510, 527 (2003). While a cursory investigation may be sufficient, a reviewing court must consider the reasonableness of the investigation that supported that strategy. Strickland, 466 U.S. at 691. Al knew of Defendants alibi claim but Al failed completely to investigate this potential defense. Als failure to investigate constituted deficient performance. In light of the other evidence of guilt (eyewitness identifications, security camera video), however, Defendant may not be able to establish prejudice. There is an argument to be made on either side. Failure to communicate plea offer The prosecutor made a plea bargain offer to Al. Al rejected the offer without communicating it to Defendant or seeking Defendants input. An attorney has a duty to consult with the client regarding important decisions, including questions of overarching defense strategy. Florida v. Nixon, 543 U.S. 175, 187 (2004); Strickland, 466 U.S. at 688. There are decisions---regarding the exercise or waiver of basic trial rights---that are of such importance that counsel cannot make them on behalf of the defendant. Nixon, 543 U.S. at 187. The defendant has the ultimate authority to determine whether to plead guilty, waive a jury, testify in his or her own behalf, or take an appeal. Jones v. Barnes, 463 U.S. 745, 751 (1983). For these significant decisions, an attorney must both consult with the defendant and obtain consent to the recommended course of action. Nixon, 543 U.S. at 187. An attorneys failure to convey a plea offer to the client constitutes deficient performance. See Arredondo v. United States, 178 F.3d 778 (6th Cir.1999); United States v. Blaylock, 20 F.3d 1458 (9th Cir.1994); United States v. Rodriguez, 929 F.2d 747 (1st Cir.1991);

DISCUSSION FOR QUESTION 7 Page 2

Johnson v. Duckworth, 793 F.2d 898 (7th Cir.1986); United States ex rel. Caruso v. Zelinsky, 689 F.2d 435 (3d Cir.1982); see also ABA Standards for Criminal Justice: Prosecution Function and Defense Function 4-6.2(b) (3d ed. 1993)(Defense counsel should promptly communicate and explain to the accused all significant plea proposals made by the prosecutor.). Al received a plea bargain offer from the prosecution. Al should have communicated that offer to Defendant. Whether to plead guilty is a decision of such importance that Al could not make it on behalf of Defendant. Nixon, 543 U.S. at 187. Defendant had the ultimate authority to determine whether to plead guilty. Barnes, 463 U.S. at 751. For this significant decision, Al should have both consulted with Defendant and obtained consent to the recommended course of action. Nixon, 543 U.S. at 187. Als failure to communicate the plea offer to Defendant satisfies the deficient performance prong of the ineffective assistance of counsel test. Failing to communicate a plea offer to a defendant constitutes prejudice if there is a reasonable probability that the defendant would have accepted the offer if it had been timely communicated. See United States v. Blaylock, 20 F.3d at 1466-67. Defendant cannot establish prejudice. Defendant maintained his innocence from the time he was stopped until he asked for counsel for an appeal. In light of Defendants conduct before, during, and after the trial, Defendant cannot establish prejudice from Als deficient performance. Therefore, Defendant was not denied the right to effective assistance of counsel by Als failure to communicate the plea bargain offer to him. Failure to appeal A criminal defendant has the right to the effective assistance of counsel in a direct appeal of his conviction. Evitts v. Lucey, 469 U.S. 387, 394 (1985). [A] lawyer who disregards specific instructions from the defendant to file a notice of appeal acts in a manner that is professionally unreasonable. Roe v. Flores-Ortega, 528 U.S. 470, 477 (2000). Counsel's failure cannot be considered a strategic decision. Id. Thus, an attorneys failure to file a notice of appeal after his client directs him to do so constitutes deficient performance. In such a case, the appellant is not required to demonstrate that his appellate claims are meritorious, because the prejudice resulting from the failure to file a notice of appeal is not in the outcome of the proceeding, but in the forfeiture of the proceeding itself. Flores-Ortega, 528 U.S. at 483. Accordingly, the defendant need not show a likelihood of success on appeal to prevail on an ineffective assistance of counsel claim based on counsels failure to perfect an appeal. Rodriquez v. United States, 395 U.S. 327, 330 (1969); see also United States v. Snitz, 342 F.3d 1154 (10th Cir.2003).

DISCUSSION FOR QUESTION 7 Page 3

Rather, to satisfy the prejudice prong of the ineffective assistance of counsel analysis in this context, the defendant need only establish that there is a reasonable probability that, but for counsels deficient performance, he would have timely appealed. Evidence of nonfrivolous grounds for appeal or the defendants prompt request for counsel to prosecute the appeal are highly relevant. Flores-Ortega, 528 U.S. at 486. The facts indicate that Defendant told the judge at the sentencing hearing that he wanted to appeal. The court appointed Carl Counselor to represent Defendant on appeal, and Carl met with Defendant who directed Carl to file a notice of appeal on his behalf. However, after Carl reviewed Als trial notes, he concluded there were no meritorious appellate issues, and did not file a notice of appeal. Carl acted in a professionally unreasonable manner by failing to follow Defendants express instructions to pursue an appeal. Defendant can thus satisfy the deficient performance prong of the ineffective assistance of counsel analysis. Defendant can also satisfy the prejudice prong, because he made a prompt request for appellate counsel by indicating at the sentencing hearing that he intended to appeal and directed Carl to file an appeal on his behalf. These facts demonstrate that, but for Carls deficient performance, Defendant would have filed a timely appeal.

2/08

COLORADO SUPREME COURT Board of Law Examiners

FEBRUARY 2008 BAR EXAM Regrade

ESSAY Q7
ISSUE

SEAT
POINTS AWARDED

1. 2. 3. 4. 5. 6. 7. 8. 9.

Recognition that the Sixth Amendment guarantees the right to effective assistance of counsel. Violation of effective assistance of counsel requires defendant show his counsel's performance was deficient, and that resulted in prejudice. Counsel's performance is judged by an objective standard of reasonableness. Al's failure to investigate alibi amounts to deficient performance. It's arguable whether Al's failure to investigate defendant's alibi prejudiced defendant. Al's failure to communicate plea offer amounts to deficient performance. It's unlikely that Al's failure to communicate the plea offer prejudiced defendant in view of defendant's consistent protestations of innocence. Carl's failure to file notice of appeal amounts to deficient performance. Carl's failure to file notice of appeal did prejudice defendant by denying him of right to appeal.

1. 2. 3. 4. 5. 6. 7. 8. 9.

4362060360

BLE Gradesheet v2.1

page 1 of 1

QUESTION 8 Lisa rented a house to Tim for a period of three years with a written lease specifying monthly rental payments. Shortly after moving in, Tim purchased and installed, with Lisas permission, a window air conditioning unit and a new in-wall fireplace. The day after the lease expired, Lisa called Tim reminding him that he needed to move out. She also told Tim that a friend wanted to use the house the following week to film a TV commercial and was willing to pay $1,500, but only if the house was vacant. Tim did not vacate by the following week, so Lisas friend chose to film the TV commercial elsewhere. Several weeks later, Tim called Lisa and told her that he had mailed her a rent check and wished to remain in the house.

QUESTION: Discuss Lisas legal rights and remedies regarding Tims ongoing occupancy of the house, and whether Tim, if he must vacate the house, can legally take the air conditioning unit and fireplace he purchased and installed.

2/08

DISCUSSION FOR QUESTION 8 1. Lisas legal rights and remedies regarding Tims occupancy. Once the lease term expired, Tim no longer had any right of possession in the house and a tenancy-at-sufferance arises. See FJK Assocs. v. Karkoski, 725 A.2d 991, 993 (Conn. App. 1999); 49 Am.Jur.2d Landlord and Tenant 284 (2006). Applicants should also receive credit if they describe Tim as a holdover tenant. See Restatement (Second) of Property, Landlord and Tenant, 14.1 (1976) (describing tenant-at-sufferance as tenant improperly holding over.) During this period, Tim is still responsible for paying Lisa a reasonable rental value of the property which typically will be the rental rate under the expired lease. See Mack v. Fennell, 171 A.2d 844, 846 (Pa. 1961) (tenant is liable for use and occupancy during such interval); Restatement (Second) of Property, Landlord and Tenant, 14.5 (1976). When faced with a tenancy-at-sufferance, Lisa has two options. First, she can treat Tim as a trespasser and utilize available common law or statutory remedies, including a wrongful detainer action, to remove or evict Tim from the property. See id.; Bryan v. Big Two Mile Gas Co., 577 S.E.2d 258, 267 (W. Va. 2001); 49 Am.Jur.2d Landlord and Tenant 273-274 (2006). Under this option, Lisa can also recover damages proximately resulting from Tims wrongful withholding of possession. See 49 Am.Jur.2d Landlord and Tenant 277-278 (2006); Restatement (Second) of Property, Landlord and Tenant, 14.6 (1976). In this case, it appears that Lisas damages would include $1500 for the lost opportunity to rent the house to her friend for the TV commercial. Alternatively, Lisa has the unilateral option of binding Tim to a new periodic tenancy. See Restatement (Second)of Property, Landlord and Tenant, 14.4 (1976). The term of the periodic tenancy can be agreed upon by the parties, but absent such an agreement, courts will look to the terms of the original lease. If, as here, the lease term exceeds one year, some authorities indicate that a year-to-year tenancy is created. See Sinclair Refining Co. v. Shakespeare, 175 P.2d 389, 391 (Colo. 1946); 49 Am.Jur.2d Landlord and Tenant 286 (2006). However, other authority provides that if rent under the expired lease was computed on a monthly basis, a month-to month periodic tenancy is created. See Restatement (Second) of Property, Landlord and Tenant, 14.4, comment (f) (1976); Roth v. Dillavou, 835 N.E.2d 425, 430 (Ill. App. 2005) (acceptance of monthly rental payments by the landlord will generally create a month-to-month tenancy). 2. If Tim vacates the house, can he legally take the air conditioning unit and fireplace? Whether Tim can legally remove these items from the house when he leaves depends on whether the items have retained their status as personal property or, instead, have become fixtures to the real property. A fixture is former personal property that is so affixed or connected with real property that it is considered to be part of the real property. See 35A Am.Jur.2d Fixtures 1 (2001).

DISCUSSION FOR QUESTION 8 Page Two

In landlord/tenant scenarios, the most important factor in determining whether an item of personal property has become a fixture is whether the owner of the personal property (the tenant) intended for the item to become part of the real property. See 8 Powell on Real Property 57.04[4] (2001); 35A Am.Jur.2d Fixtures 13 (2001); Hartberg v. Am. Founders' Sec. Co., 249 N.W. 48, 49 (Wis. 1933). An express agreement between the landlord and tenant regarding the status of the item will control. See Alexander v. Cooper, 843 S.W.2d 644, 646 (Tex. App. 1992). However, absent such an agreement (and in the present case), a court will look to various factors including (1) whether the nature of the item makes it essential to the use of the real property, (2) the manner or mode of attachment of the item, (3) whether the item is specially adapted to the real property, and (4) whether removal of the item will cause damage to the real property. See 8 Powell on Real Property 57.05[5][b] (2001). In landlord/tenant cases, there is often a presumption that a tenant would not intend to make such a donation to the property owner. See 8 Powell on Real Property 57.05[2][b] (2001); see also C.J.S. Fixtures 54 (2004). Applying the above listed factors, the window air conditioning unit that Tim installed would likely not be deemed a fixture and, therefore, may be removed by Tim. See Bay State York Co. v. Marvix, Inc., 119 N.E.2d 727, 730 (Mass. 1954) (detachable air conditioning units held to be removable by tenant). In contrast, the in-wall fireplace would appear to be more substantially attached to, and specially adapted for, Lisas house and removal would probably result in damage to the house walls. Thus, it is likely to be deemed a fixture and not removable. See Wells v. Clowers Const. Co., 476 So.2d 105, 106 (Ala. 1985) (once affixed to a house, a fireplace becomes as much a part of that house as the four walls); see also 35A Am.Jur.2d Fixtures 80 (2001).

2/08

COLORADO SUPREME COURT Board of Law Examiners

FEBRUARY 2008 BAR EXAM Regrade

ESSAY Q8
ISSUE

SEAT
POINTS AWARDED

Lisa's Legal Rights Regarding Tim's Occupancy 1. 2. 3. 4. 5. 6. Recognition of "tenancy-at-sufferance" or Tim as "holdover" tenant . As holdover/tenant-at-sufferance, Tim is still liable for reasonable value of use (rent). Lisa's first option (treat Tim as trespasser and seek eviction/wrongful detainer/removal). Can recover damages for TV commercial. Lisa's second option (bind Tim to new periodic tenancy). Term of tenancy is either year-to-year or month-to-month. 1. 2. 3. 4. 5. 6.

Lisa's Legal Rights Regarding Tim's Occupancy 7. 8. 9. Depends upon whether items are "fixtures." Focus is on intention of tenant. Intent factors: 9a. 9b. 9c. 9d. 9e. 10. 11. 12. Prior Agreement? Essential to use? Degree of attachment/ease of removal. Specially adapted. Causes damage? 9a. 9b. 9c. 9d. 9e. 10. 11. 12. 7. 8.

General presumption favoring tenants. Air conditioner (not a fixture/Tim may remove). Fireplace (a fixture/Tim cannot remove).

8153061464

BLE Gradesheet v2.1

page 1 of 1

QUESTION 9 As she was nearing death in the hospital, Jane (a widow) phoned Friend and stated: Im going to dictate a will to you and ask you to type it out and sign it for me. Friend agreed and Jane dictated the following to her over the telephone: I, Jane, make this my last will and testament. I want my sister, Susie, and my two surviving brothers, Ben and Jerry, to have everything, share and share alike. Susie will be executor of my estate. After Friend read back to Jane what she had typed, Jane instructed: OK, thats fine. Please print it out, sign it on my behalf, and keep it in a safe place. After hanging up the phone, Friend printed the will and signed Janes name to it as Jane had directed. Friends husband and daughter signed as witnesses. Jane died the following day. The total value of Janes estate is $300,000. She is survived by siblings Susie, Ben, and Jerry, and her sons, Sam and Sal. Her only daughter, Dora, predeceased Jane years before. Dora has two sons, David and Harry. Sam had no children, and Sal has two daughters, Thelma and Louise. A few years before she died, Jane had given Dora $100,000. Jane enclosed the following letter when she sent the money to Dora: I know you are in desperate need of this money now, so Im giving it to you now and will deduct it from your inheritance later. Several months before Jane died, Sal won the lottery. Shortly thereafter, Sal sent his mother the following note: Dear Mother, As you know, Im now well off financially and dont need whatever I might inherit from you. I would rather that you think of the rest of our family and not consider me in your estate planning. Love, Sal. Sals note was found among Janes effects following her death.

QUESTION: Discuss how Janes estate will be distributed. Assume the Uniform Probate Code is in effect in the jurisdiction where the will is to be probated. Also, assume Jane was competent at the time she dictated her will.

2/08

DISCUSSION FOR QUESTION 9 Did Jane execute a valid will? Whether Janes will is valid will be determined by UPC 2-502(a) and (b). Those sections read: (a) a will must be (1) in writing; (2) signed by the testator or in the testator's name by some other individual in the testator's conscious presence and by the testator's direction; and (3) signed by at least two individuals, each of whom signed within a reasonable time after having witnessed either the signing of the will as described in (2) or the testator's acknowledgment of that signature or acknowledgment of the will; (b) a will that does not comply with subsection (a) is valid as a holographic will, whether or not witnessed, if the signature and material portions of the document are in the testator's handwriting. Janes will was in writing and signed by two witnesses. In addition, the will was signed by Friend at Janes direction, but it was not signed in the testators conscious presence as required by section 2-502(a)(2). Signing [by another person] is sufficient if it was done in the conscious presence, i.e., within the range of the testators senses such as hearing; the signing need not have occurred within the testators line of sight. UPC 2-502 (comment). Rather, Friend signed the will in another location and after ending her telephone conversation with Jane. Because the signing occurred outside of range of Janes senses, the will does not meet the requirements of section 2502(a). As the material portions of the will are not in Janes handwriting, it is not a valid holographic will. Accordingly, the will is invalid, and Jane has died intestate. Did Jane make an advancement to Dora? According to UPC 2-109(a): If an individual dies intestate as to all or a portion of his or her estate, property the decedent gave during the decedents lifetime to an individual who, at the decedents death, is an heir is treated as an advancement against the heirs intestate share only if ... the decedent declared in a contemporaneous writing or the heir acknowledged in writing that the gift is an advancement. The letter that Jane sent to Dora with the check stated, I know you are in desperate need of this money, so Im giving it to you now and will deduct it from your inheritance later. This makes clear that Jane intended to the gift of $100,000 to be an advancement to Dora. However, Dora predeceased Jane. If the recipient of the property fails to survive the decedent, the property is not taken into account in computing the division and distribution of the decedents intestate estate, unless the decedents contemporaneous writing provides otherwise. UPC 2-109(c). Therefore, the amount of the advancement will not be deducted from the intestate shares of Doras sons, David and Harry.

DISCUSSION FOR QUESTION 9 Page Two Has Sal disclaimed his interest in Janes intestate estate? According to the Uniform Disclaimer of Property Interests Act (UDPIA), [a] person may disclaim, in whole or in part, any interest in or power over property. UDPIA 2-1105(a)(formerly UPC 2-801): To be effective, a disclaimer must be a writing or other record, declare the disclaimer, describe the interest or power disclaimed, be signed by the person making the disclaimer, and be delivered or filed [with the decedent estates personal representative or a court having jurisdiction to appoint a personal representative]. UDPIA 2-1105(c); see also UDPIA 2-1112(c)(delivery or filing). The disclaimer takes effect ... if the interest arose under the law of intestate succession, as of the time of the intestates death. UDPIA 2-1106(b)(1). The disclaimed interest passes ... as if the disclaimant had died immediately before the time of distribution. UDPIA 2-1106(b)(2). Sals letter to Jane stating that he did not need whatever I might inherit from you and that he would rather that you think of the rest of our family and not consider me in your estate planning will serve as a disclaimer of his interest in Janes intestate estate. The disclaimer, which was in a written note and signed by Sal, unambiguously disclaimed any right of inheritance. The disclaimer was delivered to Jane and was found with Janes effects. The disclaimer took effect upon Janes death and Sals interest in the estate will pass by representation to his daughters, Thelma and Louise. How will Janes intestate estate be distributed? The total value of Janes estate is $300,000. According to UPC 2-101(a), any part of a decendents estate not effectively disposed of by will passes by intestate succession to the decedents heirs. UPC 2-103(1) further directs that the intestate estate if there is no surviving spouse, passes ... to the decedents descendants by representation. When the decedents estate passes by representation, UPC 2-106(6) instructs: The estate ... is divided into as many equal shares as there are (i) surviving descendants in the generation nearest to the decedent which contains one or more surviving descendants and (ii) deceased descendants in the same generation who left surviving descendants, if any. Each surviving descendant in the nearest generation is allocated one share. The remaining shares, if any, are combined and then divided in the same manner among the surviving descendants of the deceased descendants as if the surviving descendants who were allocated a share and their surviving descendants had predeceased the decedent.

DISCUSSION FOR QUESTION 9 Page Three The $300,000 will be distributed by representation by dividing it into three equal shares representing the surviving descendants, Sam and Sal, and the predeceased descendant, Dora, as they are in the generation nearest to Jane containing one or more surviving descendants. Sam will receive $100,000. Due to his disclaimer, Sal will be treated as if he predeceased Jane. See UDPIA 21106(b)(3)(A). Thus, the remaining shares of Dora and Sal will be combined, and the total amount of $200,000 will be divided into equal shares and distributed by representation to Doras sons, David and Harry, and to Sals daughters, Thelma and Louise, with each receiving $50,000. Siblings Susie, Ben, and Jerry receive nothing.

2/08

COLORADO SUPREME COURT Board of Law Examiners

FEBRUARY 2008 BAR EXAM Regrade

ESSAY Q9
ISSUE

SEAT
POINTS AWARDED

1.

To be valid, a will must be (1) in writing; (2) signed by the testator; and (3) signed by at least two others, each of whom signed within a reasonable time after having witnessed either the signing of the will or the testator's acknowledgment of that signature or acknowledgment of the will. Even though Jane's will was in writing and signed by two witnesses, it is not a valid will because it was not signed by her or by another within her conscious presence. Because Jane's will is invalid, her estate will pass to her descendants by intestate succession. Property Jane gave her daughter Dora during her lifetime may be treated as an advancement. Because Jane declared in a contemporaneous writing that the gift was an advancement, such gift will therefore be counted against Dora's share. Because Dora failed to survive Jane, the advancement is not taken into account in computing the division and distribution of Jane's intestate estate to Dora's sons, David and Harry. A person may disclaim any interest in property if it is in writing, describes the interest disclaimed, is signed by the disclaimant, and is delivered to the decedent estate's personal representative. Sal's note to Jane was a disclaimer of interest in Jane's intestate estate which took effect upon Jane's death so that Sal's interest in the estate will pass by representation to his heirs, Thelma and Louise. Son Sam receives $100,000 (1/3). Nephews David and Harry receive $50,000(1/6) each. Nieces Thelma and Louise receive $50,000(1/6) each. Siblings Susie, Ben, and Jerry receive nothing.

1.

2. 3. 4a. 4b. 4c. 5.

2. 3. 4a. 4b. 4c. 5.

6.

6.

7. 8a. 8b. 9.

7. 8a. 8b. 9.

0174062684

BLE Gradesheet v2.1

page 1 of 1

QUESTION 1 Al and Bob were next-door neighbors and music afficionados. They had formerly been best friends but had feuded for years over who had the best grand piano. Al decided to settle matters by destroying Bobs piano. One Saturday night, Al waited for Bob to leave for a concert. Al entered Bobs home through an unlocked side door, approached Bobs grand piano, and gave it a few whacks with an axe. Al then left through the same door he entered. When Bob returned home, he found his piano terribly damaged. Bob realized that only Al would have done such a thing. Bob drank half a bottle of whiskey to give himself a good excuse for what he was about to do. He then walked out of his home and headed to Als house to give Al a beating. Because he was drunk, however, Bob walked past Als house and instead entered Carls house by breaking a glass back door. Carl, startled by the noise, met Bob in the living room with a firm shove, causing Bob to fall to the floor. Carl then recognized Bob and, realizing he was drunk, escorted him to the front door and out of his house. Bob walked back toward his house and, on the way, passed Als house. He looked in the front window and saw Als grand piano. Bob decided he would sneak into the house and steal the piano. Bob broke the front window of the house, climbed through the broken glass, and attempted to move the piano. After vainly attempting to move the piano for several minutes, Bob gave up, left Als home, and returned to his own home.

QUESTION: Discuss the criminal charges that may be brought against Bob and any possible defenses he may have to those charges.

7/08

DISCUSSION FOR QUESTION 1 Bob can be charged with burglary of both Carls and Als homes and attempted larceny. He would raise the defense of voluntary intoxication as to all crimes. I. Burglary Under the common law, burglary is the unlawful breaking and entering of a dwelling of another at night with intent to commit a felony therein. See Perkins v. State, 788 So.2d 826 (Miss. App. 2001); State v. Surcey, 139 N.C. App. 432, 533 S.E.2d 479 (2000); State v. Kyle, 333 N.C. 687, 430 S.E.2d 412 (1993); Wayne R. LaFave and Austin W. Scott Jr., Substantive Criminal Law, Vol II 8.13 at 464 (1986); Rollin M. Perkins, Perkins on Criminal Law, 2d.Ed. at 192 (1969). A. Burglary of Carls house Bob entered Carls house at night by breaking a glass back door. He thought he was in Als house and entered the house with the intent of committing the common law felony of battery against Al. The fact that Bob entered the wrong house does not undermine the conclusion that he committed burglary when he broke into Carls house. See State v. Jordan, 651 S.E.2d 917 (N.C.App. 2007); Com. v. Brzezinski, 2007 WL 4967503 (Pa.Com.Pl. 2007). Moreover, the actual commission of the intended felony is not an essential element of the crime of burglary. See State v. Jordan, 651 S.E.2d 917 (N.C.App. 2007); State v. Brady, 299 N.C. 547, 564, 264 S.E.2d 66, 76 (1980)(a burglary has been committed even though, after entering the house, the accused abandons his intent to commit the designated felony). Thus, Bob is guilty of burglary even though he did not commit the intended felony (battery against Al) while in Carls house. B. Burglary of Als house Bob entered Als house through a window at night with the intent to steal his piano. Again, the fact that he did not actually commit the intended felony is irrelevant. He is thus guilty of burglary of Als house. II. Attempted Larceny Initially, the better examinees might point out that the burglary of Als house was complete after Bob broke into the house, and that the attempted commission of the intended felony is a separate crime. Thus, Bob can be convicted of both burglary and attempted larceny. See State v. Cunningham, 140 N.C.App. 315, 536 S.E.2d 341 (2000). Larceny is the unlawful taking and carrying away (asportation) of the personal property of another without consent and with the intent to permanently (or for an unreasonable time) deprive the owner of his property. See Commonwealth v. Mills, 436 Mass. 387, 764 N.E.2d 854 (2001); LaFave and Scott Jr., supra Vol II 8.2 at 333; Perkins, supra at 334.

At common law, to be convicted of criminal attempt, the defendant must: (1) specifically intend to commit the crime, and (2) commit an overt act in furtherance of that intent other than mere preparation. See e.g., Grill v. State, 651 A.2d 856 (Md. 1995); State v. Mayo, 443 S.E.2d 236 (W.Va. 1994). Bob intended to take and carry away Als piano to deprive Al of it. However, Bob was unable to move the piano. Because Bob did intend to commit larceny, and he took substantial steps toward completing the offense that went beyond mere preparation (breaking the window and trying to move the piano), Bob is guilty of attempted larceny. III. Defense of Voluntary intoxication Voluntary intoxication is not a defense to the charge, but a denial that an element of the crime has been met. By contrast, when a defendant raises a defense to a charge, he admits that the elements of the offense have been met, but claims that his conduct was justified or was not criminal because of the application of the defense (for example, self-defense). See People v. Villarreal, 131 P.3d 1119, 1125 (Colo. App. 2005) (defendant was not entitled to instruction on defense of voluntary intoxication where it was inconsistent with her theory of defense that she was not the person who attacked the victim). Evidence of the defendants voluntary intoxication may be introduced when he is charged with a crime that requires purpose (intent or knowledge) to establish that the intoxication prevented him from formulating the requisite intent. State v. Robertson, 138 N.C. App. 506, 508, 531 S.E.2d 490, 492 (2000). Thus, it may negate the intent element of specific intent crimes, but not general intent crimes. See e.g., Cal. Penal Code 22(b) (Evidence of voluntary intoxication is admissible solely on the issue of whether or not the defendant actually formed a required specific intent . . . when a specific intent crime is charged); 18-1-804(1), C.R.S. (in any prosecution for an offense, evidence of intoxication of the defendant may be offered by the defendant when it is relevant to negate the existence of a specific intent if such intent is an element of the crime charged); People v. Bachofer, ___ P.3d ___, ___ (Colo. App. 2008); 2 Whartons Criminal Law (15th Edition) 111. A. Burglary of Carls House Burglary is a specific intent crime. See Velasquez v. Com., 661 S.E.2d 454 (Va. 2008); State v. Spurlock, ___ So.2d ___ (2008 WL 2190806) (La. App. 5 Cir.,2008); State v. Turnage, 660 S.E.2d 129 (N.C.App. 2008). It is thus a crime to which the voluntary intoxication defense could apply. However, Bobs assertion of the defense with respect to the charge of burglary of Carls house will be unsuccessful. The facts indicate that Bob got drunk to give himself a good excuse for what he was about to do. Thus, he had decided what he was going to do, and thus formed the intent to commit the burglary before he became intoxicated. In fact, he formed the intent to commit the crime, then got drunk for the purpose of avoiding criminal liability. Thus, Bobs intoxication did not interfere with his ability to form the intent to commit the crime and is not a defense to this charge.

B. Burglary of Als House and Attempted Larceny Attempted Larceny is also a specific intent crime to which the defense of voluntary intoxication could apply. See State v. Pascal, ___ A.2d ___, ___ (2008 WL 2581585, Conn.App. 2008)(larceny is a specific intent crime); State v. LaCroix, 911 1 A.2d 674, 678(R.I. 2006)(diminished capacity claim applies to specific intent crimes such as larceny); State v. Kiles, 175 Ariz. 358, 370, 857 P.2d 1212, 1224 (1993) ( [A]ttempt is a specific intent crime and by definition involves intentional conduct.); State v. Miller, 123 Ariz. 491, 493, 600 P.2d 1123, 1125 (App.1979) (In order to sustain a conviction for attempt there must be proof of a specific intent on the defendant's part to commit the substantive crime.).. Although Bob formed the intent to break into Als house to beat him up before becoming intoxicated, he formed the intent to break into Als house to steal his piano after he got drunk (on the way home from Carls house). Thus, his intoxication could have prevented him from forming the necessary intent to break into Als house for that purpose. Likewise, his intoxication could have interfered with his ability for form the intent to commit larceny and with his commission of overt acts in furtherance of that intent. Whether voluntary intoxication would be a successful defense to the burglary of Als house and attempted larceny charges is a question of fact for the jury to decide. See State v. Kessler, 276 Kan. 202, 210, 73 P.3d 761, 768 (2003); State v. May, 79 P.3d 795 , 795(Kan. App. 2003); State v. Robertson, supra, 128 N.C.App at 508, 531 S.E.2d at 492; Bryant v. State, 83 Md.App. 237, 574 A.2d 29, 35 (1990).

7/08

COLORADO SUPREME COURT Board of Law Examiners

JULY 2008 BAR EXAM Regrade

ESSAY Q1
ISSUE

SEAT
POINTS AWARDED

1. 2. 3. 4. 5. 6. 7. 8. 9. 10. 11. 12.

Burglary requires a breaking and entering of a dwelling of another with the intent of committing a felony therein. Larceny consists of taking and carrying away the tangible personal property of another by trespass without consent with the specific intent to permanently deprive the owner thereof. "Attempt" requires a specific intent to commit the crime and an overt act in furtherance of that intent. Charge of burglary against Bob for burglary of Carl's home. Recognition that defense of intoxication may apply. If the intent was formed after intoxication, it may be a defense to the crime. Defense of intoxication does not apply to charge of burglary against Carl's home. Charge of burglary against Bob for burglary of Al's home. Defense of intoxication may apply to charge of burglary against Al's home. Charge of larceny cannot be proved because Bob did not carry away Al's property. Charge of attempted larceny against Bob for attempting to carry away Al's piano. The impossibility of Bob lifting the piano to steal it is not a defense to Bob's attempt to steal Al's piano.

1. 2. 3. 4. 5. 6. 7. 8. 9. 10. 11. 12.

8739013236

BLE Gradesheet v2.1

page 1 of 1

QUESTION 2 John and Mary Couple jointly owned a vacation cabin in the mountains. When relations between the Couples soured, John sold the cabin to Bill Buyer without telling Mary. At closing on the property, John received a $50,000 check from Buyer made payable to John Couple and Mary Couple. John signed his own name and Marys name on the back of the check and deposited it at his bank, Deposit Bank. Deposit Bank forwarded the check for collection to Payor Bank which paid the check. John withdrew the funds collected against the check and disappeared. He never remitted any portion of the proceeds to Mary.

QUESTIONS: 1. 2. 3. Discuss any cause(s) of action that Mary may have against Payor Bank. Discuss what Mary can expect to recover in an action against Payor Bank. If Payor Bank is required to pay Mary, what rights would Payor Bank have against Deposit Bank? If Deposit Bank is required to pay Payor Bank, what rights would Deposit Bank have against John? (In answering this question, focus on only UCC remedies.)

4.

7/08

DISCUSSION FOR QUESTION 2 To properly resolve these questions the examinee must understand and apply several fundamental points of negotiable instruments law. First, an instrument made payable jointly can be negotiated only by both payees. Second, an unauthorized signature is legally ineffective as an indorsement. Third, a person who has taken up an instrument bearing an unauthorized necessary indorsement is not a holder or other person entitled to enforce the instrument. Fourth, a payor bank converts an instrument when it pays it to a person who is not entitled to enforce it. Fifth, it is a defense in a conversion action that the plaintiff had only a limited property interest in the check. Sixth, a breach of warranty action will lie against a collecting bank if it receives payment on a check bearing an unauthorized necessary indorsement.

1. Marys causes of action against Payor Bank An action in conversion will lie against a bank that paid a check which bore the genuine indorsement of only one of two joint payees because: (1) a check made payable jointly can be negotiated only with the indorsements of all the named payees; and (2) without a proper negotiation, the check was not paid to a person entitled to enforce the instrument. The Uniform Commercial Code recognizes that a bank converts a negotiable instrument when it makes a payment against it to a person who is not entitled to enforce it. UCC 3-420(a). Generally, one must be a holder or have the rights of a holder in order to be a person entitled to enforce an instrument. UCC 3-301. If an instrument is payable to the order of an identified person, that persons indorsement is necessary before any subsequent transferee can be deemed a holder. UCC 3-201(a) and (b), 1-201(20). If an instrument is payable to two or more persons jointly, indorsement by all the named payees is necessary. UCC 3-110(d), see also, 3-420, official comment 1. See generally American National Bank v. First National Bank, 277 P.2d 951 (Colo. 1954); Orr Construction Co. v. Ready Mix Concrete Co.472 P.2d 193 (Colo. App. 1970). In order to indorse an instrument, the named payees must sign it. UCC 3-204. An unauthorized signature, however, is ineffective. UCC 3-403(b). It operates only as the signature of the unauthorized signer, not the person named. Id. In the question, Buyer made a check payable to John Couple and Mary Couple. Both signatures were necessary to negotiate the instrument. Johns forgery of Marys signature was wholly inoperative as Marys signature. As a result, Deposit Savings Bank was not a person entitled to enforce the instrument. Payor Bank thus converted the check when it paid it to a person not entitled to enforce it. 2. What can Mary Expect to recover in an action against Payor Bank? In an action for conversion, a joint payee of an instrument may recover the value of the joint payees interest in the instrument. The Uniform Commercial Code applies the law applicable to the conversion of personal property to cases involving the conversion of instruments. A co-owner of personal property is entitled to damages for conversion only to the extent of their interest. U.C.C. 3-420(b) Thus, an owner of one-half of a car was entitled to

damages amounting to one-half its value. Commonwealth v. Boviard, 95 A.2d 173, 373 Pa. 47 (1953). The same principle is applicable to negotiable instruments. A drawee of a check who has converted an instrument by paying it on the indorsement of only one of two joint payees may establish that the plaintiff had only a limited interest in the check. See e.g., Lund v. Chemical Bank, 19 U.C.C. Rep. Serv. 2d 151, 170 (S.D.N.Y. 1992)(A co-payee may recover the full face amount of a check bearing his forged indorsement only if the drawee fails to establish that the payee was entitled to less than the full amount.). If Mary owned half the cabin, she was entitled to half the purchase price. U.C.C. 3-420(b)

3. What rights does Payor Bank have against Deposit Bank? A presenting bank warrants to a payor bank that it is a person entitled to enforce the instrument presented. The warranty provisions of articles 3 and 4 of the Code serve as one of the chief mechanisms the law employs to allocate loss due to forgeries and other unauthorized signatures. UCC 3-416, 3-417, 4-207, 4-208. One of the primary warranties every transferor of a negotiable instrument makes is the warranty of good title, a warranty that all indorsements are genuine. A presenting bank who receives payment for an instrument warrants it is a person entitled to enforce the instrument. UCC 4-208(a)(1)(all the other warranty provisions cited previously set forth the same guaranty). One cannot be a person entitled to enforce an instrument if it has an unauthorized signature for a necessary indorsement. Deposit Bank warranted to Payor Bank that it was a person entitled to enforce the check Buyer drew in payment for the cabin. A bank that accepts and pays a check with an unauthorized or forged indorsement warrants to subsequent transferees the validity of that indorsement and is liable on that warranty. Vectra Bank of Englewood v. Bank Western, 890 P. 2d 259 (Colo.App.1995); Thieme v. Seattle-First National Bank, 502 P.2d 1240 (1972). Deposit Bank was not a person entitled to enforce the check and it therefore breached its presentment warranty. Deposit Bank was not a holder of the instrument. Payor Bank may recover one-half the check from Deposit Bank. See U.C.C. 3-420.

4. What rights does Deposit Bank have against John? For the reasons discussed above, Marys signature was needed to negotiate the check. She was a joint payee of the instrument. See U.C.C. 3-204. John had no authority to sign her name, thus his signing was legally ineffective. U.C.C. 3-403(b). John warranted to Deposit Bank that all endorsement signatures were valid. Deposit Bank may recover from John as the warrantor all damages for breach of warranty in an amount equal to the loss suffered as a result of the breach. U.C.C. 3-416(b). Therefore, Deposit Bank may recover one-half of the amount of the check from John on his false indorsement.

7/08

COLORADO SUPREME COURT Board of Law Examiners

JULY 2008 BAR EXAM Regrade

ESSAY Q2
ISSUE

SEAT
POINTS AWARDED

1. 2. 3. 4. 5. 6. 7. 8. 9. 10. 11. 12. 13.

A negotiable instrument means a written and signed unconditional promise, to pay a fixed amount of money, payable to order or to bearer, on demand or at a definite time. An instrument made payable jointly can be negotiated only by both payees. An unauthorized signature is legally ineffective as an endorsement. A person who has taken up an instrument bearing an unauthorized necessary endorsement is not a holder or other person entitled to enforce the instrument. A payor bank converts an instrument when it pays it to a person who is not entitled to enforce it. Mary has a cause of action against payor bank. A co-owner of personal property is entitled to damages for conversion only to the extent of their interest. Mary may recover one half of the amount of the check from Payor Bank. A cause of action for breach of warranty will lie against a collecting bank if it receives payment on a check bearing unauthorized necessary endorsement. Payor bank has a cause of action against deposit bank. Recovery is limited to one half the amount of the check. Deposit Bank has a cause of action against John. Recovery is limited to one half the amount of the check.

1. 2. 3. 4. 5. 6. 7. 8. 9. 10. 11. 12. 13.

2463016260

BLE Gradesheet v2.1

page 1 of 1

QUESTION 3 Peter is an avid baseball and football card collector. Peters penchant for collecting these cards is well-known throughout the community. Peter recently hired Andy to assist him in finding and purchasing sports cards. Peter entered into an agreement with Andy, agreeing to pay Andy $50 per week. In exchange, Andy agreed to attend at least three sports card memorabilia shows per month to look for cards for Peter. The agreement further provided that Andy was only authorized to buy baseball cards and that each card purchased must be individually priced at $100 or less. Shortly after being hired, Andy attended a local sports memorabilia show where he met Sam, a card dealer, at the show. He told Sam that he was authorized to make purchases on Peters behalf. Sam was skeptical of Andys claims and called Peter to confirm what he had been told by Andy. Peter explained to Sam that Andy was only authorized to purchase sports cards that cost $100 or less. Thereafter, Andy and Sam engaged in a discussion involving the purchase of five baseball cards. Sam was willing to sell each for $100, but Andy thought the price was too high. Sam offered to throw in a sixth baseball card at no additional charge. Andy agreed to the deal and received five baseball cards for $500 and a sixth for free. Andy then purchased another baseball card on behalf of Peter for $125. Finally, Andy purchased a football card on behalf of Peter for $80. Sam allowed Andy to take possession of the seven baseball cards and one football card, agreeing to submit the bill to Peter. Andy delivered to Peter the five baseball cards which he had purchased for $500, the one baseball card which he had purchased for $125, and the one football card which he had purchased for $80. Andy kept for himself the baseball card that Sam had provided at no charge and made no mention of that card to Peter. A few days later, Peter received a bill from Sam for five baseball cards at $100 each, one baseball card at $125, and one football card at $80. Peter decided he did not want to keep any of the cards which Andy purchased and refused to pay Sam.

QUESTION: Discuss whether Sam can require Peter to pay for the baseball and football cards, and whether Andy may keep the free baseball card.

7/08

DISCUSSION FOR QUESTION 3

As an initial matter, there clearly is an express agency relationship between Peter and Andy. The creation of an agency relationship arises from the agreement of the parties to the relationship. Bradley v. Farmers New World Life Ins. Co., 679 N.E.2d 1178, 1186 (Ohio 1996). Here, the agreement established an express agency relationship that identified the scope of Andys actual authority and the compensation that he was to receive. When an agency relationship exists, the principal is bound by the acts of his agent performed on the principals behalf and within the agents actual authority. Johnson v. LeBonheur Children's Medical Center, 74 S.W.3d 338, 343 (Tenn. 2002). Accordingly, there is no doubt that Andys purchase of the five $100 baseball cards on behalf of Peter is binding upon Peter and Peter is liable to Sam for these five cards. In order to establish that an agent possessed apparent authority to bind his principal, it is essential to establish that (1) the principal held the agent out to the public as possessing sufficient authority to embrace the particular act in question and (2) the person dealing with the agent knew of the facts, and acting in good faith, had reason to believe and did believe that the agent possessed the necessary authority. Logsdon v. Main-Nottingham Inv. Co., 103 Ohio App. 233, 241-2; 141 N.E.2d 216, at 223 (1956). Under this analysis, Andy possessed apparent authority to purchase the $80 football card on behalf of Peter. Peter held Andy out as his agent authorized to purchase sports cards on his behalf. Peter did not advise Sam of the limitation as to only buying baseball cards and Sam had no actual knowledge of this limitation. When an agent acts with the scope of apparent authority, the principal will be bound by the acts of the agent performed on the principal's behalf. Johnson v. LeBonheur Children's Medical Center, 74 S.W.3d at 343; James v. Alberts, 464 Mich. 12, 15, 626 N.W.2d 158, 159 (2001). Further evidence of Sams good faith is the fact that he specifically inquired of Peter as to whether Andy was acting on behalf of Peter. A third party who deals with a known agent should not blindly trust the agents statement as to the extent of the agents authority. Rather, one must use reasonable diligence and prudence to ascertain whether the agent acts within the scope of his authority. Gen. Cartage & Storage Co. v. Cox, 74 Ohio St. 284, 78 N.E. 371 (1906); ABC Outdoor Adver., Inc. v. Dolhuns Marine, Inc., 38 Wis.2d 457, 461, 157 N.W.2d 680, 682 (1968) (principal is bound by acts of agent acting within scope of apparent authority as to contracts with third parties if the third party, acting in good faith, believes and has reason to believe the agent has such authority) . For many of the reasons set forth above, Peter will not be bound by Andys attempt to purchase the $125 baseball card and Sam will not be able to enforce this sale against Peter. Sam had actual knowledge via his telephone conversation with Peter that Andy did not have authority to purchase any sports cards that cost in excess on $100. Additionally, because of this actual knowledge, Sam cannot argue that Andy possessed apparent authority. See Logsdon, 103 Ohio App. 242, 141 N.E.2d at 223. As a final matter, Peter has several claims that he can assert against Andy. One who acts as an agent for another becomes a fiduciary with respect to the matters within the scope of the agency relationship. Miles v. Perpetual Sav. & Loan Co., 388 N.E.2d 1364, 1365 (Ohio 1979).

Unless otherwise agreed, an agent is subject to a duty to his principal to act solely for the benefit of the principal in all matters connected with his agency." Plummer v. Estate of Plummer, 51 S.W.3d 840, 842 (Tex. App 2001); MDM Group Assoc., Inc. v. CX Reinsurance Co. Ltd., 165 P.3d 882 (Colo. App. 2007) (quoting Restatement (Third) of Agency, 8.01 (2006), that [a]n agent has a fiduciary duty to act loyally for the principals benefit in all matters connected with the agency relationship). [A]n agent is not entitled to avail himself of any advantage that his position may give him to profit beyond the agreed compensation for his service. . . . He will be required to account to his principal for any gift, gratuity, or benefit received by him in violation of his duty, or any interest acquired adverse to his principal without full disclosure, though it does not appear that the principal has suffered any actual loss by fraud or otherwise." Texana Oil & Refining Co. v. Belchic, 150 La. 88, 102, 90 So. 522, 527 (1922). Clearly, when Andy kept the sixth free baseball card and did not disclose its existence to Peter, he breached the obligations identified above. Moreover, when Andy intentionally exceeded the scope of his actual authority, this likewise violated his fiduciary obligation to his principal.

COLORADO SUPREME COURT Board of Law Examiners

JULY 2008 BAR EXAM Regrade

ESSAY Q3
ISSUE

SEAT
POINTS AWARDED

1. 2.

A principal-agent relationship exists between Peter and Andy because they each consented to the relationship. Actual authority may be: 2a. 2b. Express Implied

1.

2a. 2b. 3. 4.

3. 4. 5.

Sam has valid claim against Peter for the five $100 baseball cards. Sam has a valid claim against Peter for the $80 football card based upon Andy's apparent authority. Agency may also exist from apparent authority; the elements are: 5a. 5b. The principal held the agent out to the public as possessing sufficient authority to embrace the particular act in question. The person dealing with the agent knew of the facts, and acting in good faith, had reason to believe and did believe that the agent possessed the necessary authority.

5a. 5b. 6. 7. 8.

6. 7. 8.

Peter has a defense to Sam's attempt to enforce the sale of the $125 baseball card based upon the express limitation of authority made known to Sam. Keeping the free baseball card without disclosing that information to Peter constituted a breach of the agent's fiduciary duties. Andy intentionally exceeded the scope of his authority (Purchase of Football Card and $125 card).

1213019445

BLE Gradesheet v2.1

page 1 of 1

QUESTION 4 While driving home from work last month, Paul Plaintiff was involved in an auto accident with Doris Defendant. Defendant had entered an intersection as the light was changing from yellow to red and ran into Plaintiff. Charlie Officer, a police officer, arrived at the accident scene about twenty minutes later and wrote a report. Plaintiff, alleging negligence, brought a civil lawsuit against Defendant for damages to his car.

QUESTIONS: Applying the Federal Rules of Evidence, discuss the admissibility of the following evidence offered at trial. 1. Defendant called Officer to testify that she told Officer at the accident scene that she had entered the intersection on a green light. Plaintiff called Officer to testify that Defendants drivers license, which Officer examined at the accident scene, restricts Defendant to driving with corrective lenses and that Defendant was not wearing them that day. Defendant offered Officers police report into evidence. The report contains a description of the accident scene and records a statement from Plaintiff stating that he had been heading home from a bar when the accident occurred. Officer testified that he prepares such reports routinely at accident scenes, and is always careful to record the facts and statements accurately. After Plaintiff testified, Defendant offered Officers testimony that Plaintiff was convicted seven years ago for misdemeanor fraud.

2.

3.

4.

7/08

DISCUSSION FOR QUESTION 4

The Federal Rules of Evidence (F.R.E.) will determine the admissibility of each item of evidence. 1. Defendants Statement

Officers testimony that Defendant told him at the accident scene that she entered the intersection on a green light is hearsay. Hearsay is an out-of-court statement offered for the truth of the matter asserted therein. F.R.E. 802. Here, the statement is being offered for its truth to prove that the light was green, thus eliminating Defendants liability. Hearsay is inadmissible unless the particular hearsay satisfies one of the exceptions to the hearsay prohibition. Here, the following arguments may be made. First, Defendant will argue that her statement constitutes a present sense impression. This exception applies to hearsay that describes or explains an event or condition and is made while the declarant was perceiving the event or condition, or immediately thereafter. F.R.E. 803(1). Defendant is the declarant, and although her statement does describe the event, she was not observing the event (her automobile entering the intersection) while she made the statement. Thus, she must argue that her statement was made immediately after the event. This argument will likely fail. Although courts have not identified a rigid time frame that constitutes immediately thereafter, see United States v. Blakey, 607 F.2d 779 (7th Cir. 1979) (23 minutes was immediately thereafter); Hilyer v. Howat Concrete, 578 F.2d 422, 426 n.7 (D.C. Cir. 1978) (15 minutes or more too long), the better answer in this case is that Defendant statement came too long after the event. Defendant was intimately involved in the accident, and the passage of time coupled with the police investigation gave Defendant motivation to fabricate. Next, Defendant may argue that her statement constitutes a party admission. Party admissions are not hearsay. Fischer v. Forestwood Co., Inc., 525 F.3d 972 (10th Cir. 2008). F.R.E. 801(d)(2). Although Defendants statement was made by a party to the litigation, it was not offered into evidence by Plaintiff, her party-opponent. The statement thus does not constitute an admission by a party opponent under F.R.E. 801(d)(2). Defendant may also argue that her statement is admissible for a non-hearsay purpose, as a prior consistent statement. F.R.E. 801(d)(1)(B). For this theory to be applicable, Defendant would have to testify and be subject to cross examination. Her testimony would have to be impeached as the product of a motivation to fabricate, and her statement to Officer would have to precede her motivation. There is nothing in the facts to indicate a motive to fabricate. This could only be the fact if, for example, Defendant made the statement and then an intervening motive to fabricate arose - such as Defendant being paid to change her testimony. Tome v. United States, 513 U.S. 150 (1995); United States v. Gomez, 191 Fed.Appx. 413 (6th Cir.2006). As no facts indicating such an intervening event or changed testimony are present here, the better answer is that this argument should fail

2.

Defendants Drivers License

Any evidence that tends to make a material issue more or less true is relevant. F.R.E. 402. Officers testimony that Defendants license reveals a driving restriction, and that she was without her corrective lenses, is relevant because her need for corrective lenses when driving, and her failure to wear them, would help establish Plaintiffs claim of negligence. Defendant would object to this evidence on the grounds of the best evidence rule. This rule excludes testimony about the contents of a writing, photograph or recording, and instead requires the proponent to produce the original, a duplicate, or an excuse for its non-production. F.R.E. 1002. The court would probably sustain the objection. Although Officer may testify to his observation that Defendant was not wearing corrective lenses, he may not testify to the contents of the writing on the license; he must produce the license, a duplicate, or an excuse for its non-production. 3. Police Report

Defendant offers Officers police report, which contains a description of the accident scene and records a statement from Plaintiff stating that he had been heading home from a bar when the accident occurred. Officer testified that he prepares such reports routinely at accident scenes, and is always careful to record the facts and statements accurately. Police reports are generally inadmissible in criminal cases. F.R.E. 803(8); United States v. Oates, 560 F.2d 45 (2d Cir. 1977). This is a civil case, however, so this prohibition will not apply. See Truesdale v. Klich, , 2006 WL 1460043 (N.D.Ill.,2006) (police reports, which recorded first-hand observations of testifying officers, were admissible in civil trial under the public records and reports exception to the hearsay rule); Bolduc v. United States, 265 F.Supp.2d 153 (D.Mass.,2003 (same). The evidence contains hearsay within hearsay. Officers police report, a hearsay document, contains Plaintiffs hearsay statement. Hearsay within hearsay is admissible as long as each embedded hearsay statement falls within one or more hearsay exceptions. F.R.E. 805. Plaintiffs statement constitutes a party admission under F.R.E. 801(d)(2). It is a statement by a party offered into evidence by a party-opponent, here Defendant. It may also be used to impeach Plaintiffs credibility if Plaintiff offers contradictory evidence at trial. In such an event, the testimony is only narrowly admissible for impeachment as to credibility. One could also argue that Plaintiffs statement fits within the statement against interest hearsay exception in F.R.E. 804(b)(3). This exception applies to statements that are so far contrary to the declarants interests that no reasonable person would have made them unless they were true. F.R.E. 804(b)(3). This argument would likely be unsuccessful because this exception is available only where the declarant (here, Plaintiff) is unavailable as a witness. No facts in the problem suggest that Plaintiff, a party, is not available to testify. Officers police report is a business record. It is a record made contemporaneously with the events Officer was describing (the accident scene and the statements by the participants) and

was made as a regular part of Officers business. F.R.E. 803(6). Officers testimony lays the requisite foundation for this exception to apply. Defendant may thus offer the report; it is admissible because it is a business record and the hearsay statement contained therein is a party admission. See Truesdale, supra; Bolduc, supra. 4. Plaintiffs Prior Conviction

Prior convictions may be used to impeach the testimony of a witness. F.R.E. 609. When the nature of the crime that underlies the prior conviction involves dishonesty or a false statement, that conviction is admissible to impeach even if it is a misdemeanor and without any consideration of the potential unfair prejudice from the impeachment under F.R.E. 403. F.R.E. 609(a)(2). Fraud is a crime involving dishonesty or a false statement. The conviction occurred within the last ten years, and thus the prohibition on the use of convictions more than ten years old does not apply. F.R.E. 609(b).

7/08

COLORADO SUPREME COURT Board of Law Examiners

JULY 2008 BAR EXAM Regrade

ESSAY Q4
ISSUE

SEAT
POINTS AWARDED

1. 2. 3. 4.

Hearsay is an out-of-court statement offered for the truth of the matter asserted. F.R.E. 801(c), 802. Doris's statement is hearsay because it is offered for its truth. F.R.E. 802. Doris's statement is not a present sense impression or excited utterance because it was made too long after the event was perceived. F.R.E. 803(1). Doris's statement is not an admissible party admission because although the statement was made by a party, it was not offered by a party-opponent. F.R.E.801(d)(2).

1. 2. 3. 4.

5. Doris's statement is not an admissible prior consistent statement. F.R.E.801(d)(1)(B). 5. _________________________________________________________________________________________ 6. 7. 8. Relevant evidence is evidence that tends to show that a fact is more or less likely to be true. F.R.E. 402. Paul's proffered testimony of Charlie (2) regarding Doris's license is relevant because her need for glasses and failure to wear them would be evidence of her negligence. The best evidence rule states that one may not testify about the contents of a photograph or writing, but instead must produce the original, a duplicate, or an excuse for its non-production. F.R.E. 1002. 6. 7. 8.

9. This testimony would be precluded by the best evidence rule. 9. _________________________________________________________________________________________ 10. 11. 12. Doris's proffer of Charlie's police report (3) is double hearsay because the report is hearsay and the report contains the statements of Paul, which are hearsay. Double hearsay is admissible if there is one or more hearsay exceptions applicable to each instance of hearsay. F.R.E. 805. The prohibition against police reports in criminal cases does not apply here because this is a civil case. F.R.E. 803(8)(B). 10. 11. 12.

5246022360

BLE Gradesheet v2.1

page 1 of 2

COLORADO SUPREME COURT Board of Law Examiners

JULY 2008 BAR EXAM Regrade

ESSAY Q4
ISSUE

SEAT
POINTS AWARDED

13. 14. 15. 16.

Paul's statement in the report is a party admission and is thus admissible because it is a statement of a party offered by a party-opponent. F.R.E. 801(d)(2). Paul's statement is not admissible under the "statement against interest" exception because the facts do not indicate that Paul is unavailable to testify. F.R.E. 804(b)(3). The police report falls within the business records or public records exception. F.R.E. 803(6).

13. 14. 15.

16. The report is admissible because both hearsay statements (the report and Paul's statement therein) are independently supported by exceptions. _________________________________________________________________________________________ 17. 18. 19. 20. 21. Prior convictions may be used to impeach the testimony of a witness. F.R.E. 609. When the nature of the crime that underlies the prior conviction involves dishonesty or a false statement, that conviction is admissible. It is admissible to impeach even if it is a misdemeanor and without any consideration of the potential unfair prejudice from the impeachment under F.R.E.403. F.R.E.609(a)(2). The misdemeanor conviction occurred within the last ten years, and thus the prohibition on the use of convictions more than ten years old does not apply. F.R.E. 609(b). Paul's prior conviction (4) is admissible to impeach Paul's truthfulness. 17. 18. 19. 20. 21.

1908022361

BLE Gradesheet v2.1

page 2 of 2

QUESTION 5 In February 2008, the following appeared in Scientific American: The Association for Man-Powered Flight (AFMPF) offers a prize of $100,000 to the first person who can build an aircraft which can fly a distance of one mile powered only by human muscle power. Proof of a successful entry must be submitted before May 15, 2008. Peter was told about the contest by his friend, Ronald. Upon hearing of the contest, Peter, who was already trying to develop such a craft, re-intensified his efforts on the construction of his craft called Muscle II. It had been a lifelong dream of Peter to construct a muscle-powered aircraft. His Muscle I, however, had crashed early last December. In March 2008, Peter also wrote to Frank Feather, the President of the AFMPF, saying he planned to enter his craft, Muscle II, in the competition. However, the letter was never delivered to the companys headquarters. On May 1, the following notice appeared in Scientific American: The contest sponsored by the Association for Man-Powered Flight (AFMPF) regarding muscle powered aircraft is canceled. Peter did not see this notice. On May 10, Peter tested Muscle II before a group of certified experts. The aircraft performed as required, flying the mile course in about 4 minutes. Peter then submitted proof of his success to AFMPF prior to May 15, 2008, and was the first to claim the prize. AFMPF refused to pay.

QUESTION: Discuss the claims Peter may have against AFMPF, and any defenses AFMPF may have against such claims.

7/08

DISCUSSION FOR QUESTION 5 (1) Determining Peters claims requires an analysis of whether AFMPFs announcement of the contest constitutes an offer that became binding upon AFMPF by Peters performance. An offer is the manifestation of willingness to enter into a bargain, so made as to justify another person in understanding that his assent to that bargain is invited and will conclude it. Restatement (Second) of Contracts 24; John D. Calamari & Joseph M. Perillo, The Law of Contracts 2.5, at 31 (3d ed. 1987). Advertisements of goods by display, sign, handbill, newspaper, radio or television are not ordinarily intended or understood as offers to sell. The same is true of catalogues, price lists, and circulars, even though the terms of suggested bargains may be stated in some detail. It is of course possible to make an offer by an advertisement directed to the general public, but there must ordinarily be some language of commitment or some invitation to take action without further communication. Restatement (Second) of Contracts 26, comment b. AFMPFs announcement can be construed as containing a promise, as its terms are certain and definite and an offeree is clearly identified. Lefkowitz v. Great Minneapolis Surplus Store, Inc., 251 Minn. 188, 191-192, 86 N.W.2d 689, 691 (1957) (the authorities emphasize that, where the offer is clear, definite, and explicit, and leaves nothing open for negotiation, it constitutes an offer, acceptance of which will complete the contract). Consequently, in the present case, a reasonable person in Peters position would understand that when AFMPF stated that it would award a prize to the first person that constructed and flew the craft, that it intended to enter into a bargain. See Restatement (Second) of Contracts 45 ill. 4 & 5. An offer may require acceptance to be made only by performing a specified act. Id. at 30(1). If the offer does not call for a promissory acceptance, but rather invites the offeree to accept by rendering a performance, a contract is created when the offeree tenders the invited performance. Id. 32, comment b, 45(1); Calamari, 2.22 at 113. The language and circumstances in this case make it clear that the offeree is not to bind himself in advance of performance. His promise would be worthless to the offeror, and the circumstances make it unreasonable for the offeror to expect a firm commitment from the offeree. In this case, the offer does not invite a promissory acceptance, and a promise would be ineffective as an acceptance. Offers of reward or of prizes in a contest, made to a large number of people, but to be accepted by only one, such as in this case, are classic examples of unilateral contracts the offer of which can be accepted by performance only. Id. at 32 comment b; See also 29. Acceptance of an offer is a manifestation of assent to the terms thereof made by the offeree in a manner invited or required by the offer. Restatement (Second) of Contracts 50. Where the offer requires acceptance by performance and does not invite a return promise, as in the ordinary case of an offer of a reward, a contract can be created only by the offeree's full performance. Id. 50 comment b. However, where the offeree may choose to create a contract either by making a promise or by rendering or tendering performance, the beginning of performance or a tender of part performance operates as a promise to render complete performance. Id. 32, 62. In this case Peter accepted AFMPFs offer when he built an aircraft which flew a distance of one mile powered only by human muscle power. This is true even

though Peter was already working on the craft. Unless the offeror manifests a contrary intention, an offeree who learns of an offer after he has rendered part of the performance requested by the offer may accept by completing the requested performance. Id. 51. Where an offer invites an offeree to accept by rendering a performance, no notification is necessary to make such an acceptance effective unless the offer requests such a notification. Id. 54(1); Carlill v. Carbolic Smoke Ball Co., [1893] 1 Q.B. 256 (U.K.C.A.); Rague v. New York Evening Journal Publishing Co., 164 A. D. 126, 149 N.Y.S. 668 (1914). In this case, the fact that Peters letter to AFMPF was never delivered would not prevent him from being awarded the prize money.

(2) Any defense AFMPF has depends on whether the offer could be revoked. Generally, an offer may be revoked if the revocation occurs before the offeree accepts or completes performance. An offer made by advertisement, or by a general notification, to the public, is revoked by an advertisement or general notice given publicity equal to that given to the offer before a contract has been created by acceptance of the offer. The revocation by publication is effective when published. Restatement (Second) of Contracts 46, Ill.1. However, because Peters performance had been partially tendered prior to the cancellation, AFMPFs offer had become irrevocable. Id. 45, 62, comment b. Thus, AFMPF does not have a defense against Peters claim for payment.

7/08

COLORADO SUPREME COURT Board of Law Examiners

JULY 2008 BAR EXAM Regrade

ESSAY Q5
ISSUE

SEAT
POINTS AWARDED

1. 2. 3. 4. 5. 6. 7. 8. 9.

Generally an advertisement is not an offer and is considered a "solicitation" or "invitation" for an offer rather than an actual offer. A unilateral contract may be formed, however, where (1) there is an offer to the public, and (2) performance under the offer is the only means of acceptance contemplated. Here, the advertisement in the Scientific American constitutes an offer because it promises a reward under terms that are sufficiently certain and definite. Moreover, the terms of offer contained in the advertisement contemplate that it can be accepted only by performance. Generally, an offer can be revoked if the revocation is communicated/published to the offeree in the same manner as the original offer. An offer under a unilateral contract may become irrevocable, however, once performance has begun (thereby creating an "option" contract). The advertisement withdrawing the prize offer was not effective given that Peter's performance had already begun under the offer. There is no need to give formal notice of acceptance where performance is required to accept an offer. Thus, it does not matter that Peter's note was never received by AFMPF. Peter effectively accepted the offer and formed the unilateral contract when he was the first to build and successfully fly a man powered aircraft one mile and submit proof prior to the deadline.

1. 2. 3. 4. 5. 6. 7. 8. 9.

5748160584

BLE Gradesheet v2.1

page 1 of 1

QUESTION 6 Tom Client contacted his lawyer, Andrew Attorney, and asked him to set up a corporation for his business, to be called Computer Programs, Inc. (CPI). Client was to be the sole shareholder. On January 1, 2008, Attorney drafted articles of incorporation and gave them to Client who immediately signed them. The articles read in their entirety, as follows: Articles of Incorporation These Articles of Incorporation are filed on behalf of Computer Program, Inc., whose registered agent is Andrew Attorney, Esq., 1313 Mockingbird Lane, South Park, State. ___________(signed)______________ Tom Client, Incorporator _________________________________ Additional Incorporator Dated: ____(dated)_____, 2008 Dated: _______________, 2008

Only Client signed the articles. The next day, January 2, 2008, Attorney mailed the articles of incorporation to the Secretary of State and emailed Client, saying, Good news! I have mailed the articles to the Secretary of State today. Thinking that meant that CPI was now a corporation, Client met with Landlord on January 3, 2008, and signed a lease for office space in which to house CPI. Client signed the lease Tom Client, President of CPI. The next day, January 4, 2008, the Secretary of State notified Attorney that it had rejected CPIs articles of incorporation. Attorney contacted Client with the news that CPIs articles had been rejected. That same day, Client notified Landlord that the lease was invalid because CPI had not been approved as a corporation. Instead of renting other space, Client decided to use CPI funds to purchase a single powerful computer and run CPI from his home. He used the computer for both CPI and personal business. On January 5, 2008, Client instructed Attorney to correct the articles of incorporation and resubmit them. On January 6, 2008, the Secretary of State accepted and filed the articles.

QUESTION: Assume that the state in which Client formed CPI follows the Revised Model Business Corporation Act (RMBCA). Discuss why the Secretary of State initially rejected the Articles of Incorporation and whether CPI or Client is liable to Landlord under the lease.

7/08

DISCUSSION FOR QUESTION 6 Why did the Secretary of State reject the Articles of Incorporation? According to the RMBCA 2.02(a), articles of incorporation must contain at least the following: 1. The name of the corporation, which must include a word like corporation, incorporated, company, or limited. (See also RMBCA 4.01.) 2. The number of shares the corporation is authorized to issue. 3. The name and street address of its registered agent. 4. The name and address of each incorporator. Here, the articles of incorporation were insufficient because they did not contain either the number of shares CPI was authorized to issue or the name and address of its incorporator. Although only Client signed as an incorporator, leaving the other line blank, that was not a reason for rejecting the articles of incorporation. Under RMBCA 2.01, only one incorporator is needed to form a corporation. Who Is liable for the lease, CPI or Client? Under RMBCA 2.03, a corporation comes into existence on the date the Secretary of State accepts its articles of incorporation for filing. Here, that occurred after the lease was signed; therefore, at the time Client signed the lease, CPI did not yet exist. Landlord, therefore, cannot rely on the statute to assert that CPI, as a corporate entity, is liable on the lease. Under RMBCA 2.04, an incorporator is personally responsible for liabilities he incurs on behalf of a would-be corporation if he does so knowing there was no incorporation under this Act. Here, although Client incurred liability for a non-existent corporation, he did not do so knowingly. He thought that CPI was a corporation at the time he signed the lease. Landlord might argue that it should be allowed to pierce the corporate veil and assert personal liability against Client. Personal liability, by virtue of piercing the corporate veil, is only imposed under very narrow circumstances. The facts here raise only one possibility the alter ego rule. Under the alter ego rule, a shareholder, like Client, forfeits the benefits of the corporate veil if he treats the corporation as if it were his alter ego. One way of doing so is to use the assets of the corporation for his own personal use. Fletcher on Corporations 41.10. Here, Client has done so with CPIs computer. However, the commingling of just one asset, such as the computer alone, is insufficient. Generally, the alter ego rule requires a creditor to show complete control and domination of the corporation by the shareholder and further that such control was used by shareholder for purpose of conducting a subterfuge, resulting in injustice. Id. Using just one asset as here, the computer, for both corporate and personal reasons, while unwise, is probably insufficient to establish complete control and domination. Also, there are no facts suggesting subterfuge. Rather, as explained, Client honestly (albeit

mistakenly) thought CPI was a corporation when he signed the lease. He was not attempting to misuse a corporation as a vehicle for injustice. Client may also argue that equity does not require piercing the corporate veil here because, for example, Landlord could have but did not ask for a personal guaranty. As explained, (c)ourts apply the alter ego rule with great caution and reluctance. Id. In fact, many courts require exceptional circumstances before disregarding corporate form. Id. Here, Landlord is unlikely to be able to prove the requisite exceptional circumstances needed to show that equity requires piercing CPIs corporate veil. Ultimately, however, Landlord may find relief under the common law doctrine of corporation by estoppel. Generally, under modern statutes, such as the RMBCA, the concept of corporation by estoppel has been replaced in favor of the bright line of when articles of incorporation are filed with the Secretary of State. Id., 3890. As explained, an incorporating shareholder is personally liable, under the RMBCA, for debts knowingly incurred on behalf of a nonexistent corporation. Here, though, Client was not aware the corporation did not exist. The doctrine of corporation by estoppel provides Landlord with an opportunity to estop both CPI and Client from denying that CPI didnt exist at the time Client signed the lease. Under the doctrine of corporation by estoppel, a corporation is estopped from denying its nonexistence if it has held itself out as a corporation and took advantage of its purported corporate structure. Id., 3889. The estoppel of CPI also acts to estop its incorporator, Client. Id., 3938. Here, CPI, through its incorporator Client, held itself out as a corporation. It acted as if it were a corporation. Client signed the lease for CPI as a corporation. Id., 3932. Landlord relied by accepting the signature as sufficient to bind CPI. CPI is therefore estopped from denying its nonexistence as a corporation when the lease was signed; therefore, it is liable for the rent. Once estoppel has been established, Landlord is precluded from denying CPIs existence and would, therefore, be unable to proceed against Client for personal liability. Id., 3890.

7/08

COLORADO SUPREME COURT Board of Law Examiners

JULY 2008 BAR EXAM Regrade

ESSAY Q6
ISSUE

SEAT
POINTS AWARDED

1. 2. 3. 4. 5.

Articles of incorporation were rejected because they failed to state the number of shares CPI was authorized to issue. Articles also failed to state the address of its incorporator. The signature of one incorporator is sufficient to establish a corporation. CPI did not exist until the Secretary of State accepted its articles of incorporation for filing, therefore no liability to CPI. Generally an incorporator is personally liable for liability he incurs on behalf of a would-be corporation. 5a. Here, Client signed the lease thinking CPI existed, therefore no personal liability. Client would be personally liable if Landlord can pierce the corporate veil. One reason for piercing would be if a shareholder used corporate assets as his own (alter ego rule). Generally, alter ego application requires complete control & domination by shareholder. 8a. Client's use of just one asset (CPI's computer) is probably insufficient.

1. 2. 3. 4. 5. 5a. 6. 7. 8. 8a. 9.

6. 7. 8. 9.

Landlord may be able to collect rent from CPI under the equitable doctrine of corporation by estoppel.

0229162282

BLE Gradesheet v2.1

page 1 of 1

QUESTION 7 PizzaFast, a pizza company, entered into an agreement with Dave to deliver pizzas. Under the agreement, Dave was paid per delivery, was required to wear a PizzaFast uniform, and had to display a PizzaFast sign atop his car when making deliveries. Dave was also required to comply with PizzaFast policies which included a strict, no exception policy requiring all call-in orders to be delivered within thirty minutes. PizzaFast recently notified workers that the thirtyminute deadline was of paramount importance to the company. When PizzaFast entered into the agreement with Dave, it was aware that Dave had been at fault in several recent traffic accidents. During his first two months of work for PizzaFast, Dave was at fault in two more traffic accidents while delivering pizzas. However, because Dave was willing to work long hours and always showed up for work, PizzaFast continued to retain his services. One night, Dave received a pizza order that ordinarily would have been easily deliverable within the deadline period. However, when Dave got into his car, he remembered he had forgotten to close the garage door of his house. Because his house was in the same general direction as the customers address, Dave decided to stop at his house and close the garage door. Upon retuning to his car, Dave realized he had only four minutes remaining until the delivery deadline expired. While rushing to the customers address, Dave negligently struck a vehicle driven by Paul causing damage to both vehicles. Pauls vehicle came to a rest blocking Daves car. Aware of the looming deadline, and PizzaFasts no exception policy of meeting that deadline, Dave purposefully bumped Pauls vehicle with his own to clear it out of the way (causing injury to Paul). Dave then completed the pizza delivery with only seconds to spare.

QUESTION: Discuss any claim(s) that Paul may have against PizzaFast.

7/08

DISCUSSION FOR QUESTION 7

I. Respondeat Superior/Vicarious Liability of PizzaFast Paul may assert a claim against PizzaFast based upon respondeat superior liability for Daves torts. The doctrine of respondeat superior provides that an employer may be held vicariously liable for an employee's torts when the act is committed within the course and scope of employment. To establish a defendant's liability under the doctrine, the plaintiff must show that (1) an employer-employee relationship existed and (2) the tort occurred in the course and scope of employment. See Biddle v. Sartori Mem'l Hosp., 518 N.W.2d 795, 797 (Iowa 1994); Colo. Compensation Ins. Auth. v. Jones, 131 P.3d 1074, 1079 -1080 (Colo. App. 2005). The most important factor in determining whether a worker qualifies as an employee for purposes of respondeat superior liability is the alleged employer's right to control the manner in which the work is performed. See Hodges v. City of St. Louis, 217 S.W.3d 278, 282 (Mo. 2007) (dissent), citing, Ross v. St. Louis Dairy Co., 339 Mo. 982, 994, 98 S.W.2d 717, 723 (1936); Kaplan v. U.S. Bank, 166 S.W.2d 60, 66 (Mo. App. 2003) (If there is no right to control, there is no liability.); Jones, 131 P.3d at 1080. Here, because the agreement required Dave to wear the PizzaFast uniform, display the PizzaFast sign, and comply with PizzaFast policies, control is present and an employer-employee relationship exists. An act is considered to be within the scope of employment if it is performed while the employee is engaged generally in the business of his employer, or if his act may be reasonably said to be necessary or incidental to such employment. Holmes v. Gary Goldberg & Co., 838 N.Y.S.2d 105, 106 (N.Y. App. Div. 2007); see Destefano v. Grabrian, 763 P.2d 275, 287 (Colo. 1988) (engaged in work which has been assigned by employer or is doing what is necessarily incidental to the assigned work customary within the business). Another issue is whether Daves side trip to his house constituted a frolic or detour. Such action can potentially take an employee outside of the course and scope of employment. See Lime City Mut. Ins. Assn. v. Mullins, 615 N.E.2d 305, 309 (Ohio Ct. App. 1992). However, because Daves detour was minor both in terms of time and direction, and because Dave had resumed the delivery process at the time of the tortuous conduct, the detour will probably not preclude Paul from recovering based upon respondeat superior. See Prosser & Keeton, The Law of Torts 504-05 (5th ed. 1984) (ordinarily when a frolic and detour is ended and the employee returns to his or her original route, the employee is again within the scope of employment). Finally, PizzaFast may be vicariously liable both for Daves negligence in initially striking Pauls vehicle and for his intentional tort of battery in purposely striking Pauls vehicle the second time. Employers are generally not vicariously liable for the intentional torts of their employees. See Keller v. Koca, 111 P.3d 445, 448 n.2 (Colo. 2005). However, if an employees intentional tort is motivated by an intent to serve the employer's purpose, the employer may be vicariously liable. See Maras v. Milestone, Inc., 809 N.E.2d 825, 828, (Ill. App. Ct. 2004); see also Moses v. Diocese of Colo., 863 P.2d 310, 330 n. 27 (Colo. 1993); Jones, 131 P.3d at 1080.

Here, the facts reveal that Daves motivation for intentionally bumping Pauls was to allow him to continue the delivery and adhere to the PizzaFast no exception policy of meeting the thirtyminute deadline. Thus, he was serving his employers purpose. II. Direct Liability of PizzaFast Paul may also assert a direct negligence claim against PizzaFast. In order to prevail on a negligence claim, Paul would have to establish a duty on the part of PizzaFast, a breach of that duty, proximate causation, and resulting damages. See Camalier v. Jeffries, 460 S.E.2d 133, 136 (N.C. 1995). First, Paul could assert a claim for negligent hiring or retention of Dave based upon his pre-hire and post-hire at fault accidents. See Tallahassee Furniture Co. v. Harrison, 583 So.2d 744, 759-60 (Fla. Dist. Ct. App. 1991) (employees criminal and hospital records were admissible to prove the employees unfitness for employment or retention); Petrik v. N.H. Ins. Co., 379 So.2d 1287, 1289 (Fla. Dist. Ct. App. 1979) (employee's past driving record would be admissible to show negligent hiring or employment of employee). Paul could also assert that PizzaFast was negligent in implementing and strictly enforcing a thirty minute delivery policy, because of foreseeable risk that drivers may act negligently or recklessly to meet the deadline and thereby harm others. See Weirum v. RKO Gen., Inc., 539 P.2d 36, 40 (Cal. 1975) (upholding negligence claim against radio station based upon contest that forseeably would cause listeners to drive hastily to location and disregard the demands of highway safety.) (Note: Paul could also argue that PizzaFast might be liable for punitive damages. See Wauchop v. Dominos Pizza, Inc., 832 F.Supp. 1577, 1581 (N.D. Ind. 1993) (in denying motion for summary judgment, the court held that the pizza franchisor could be liable for punitive damages for thirty-minute delivery guarantee when it consciously disregarded risks associated with the guarantee).

7/08

COLORADO SUPREME COURT Board of Law Examiners

JULY 2008 BAR EXAM Regrade

ESSAY Q7
ISSUE

SEAT
POINTS AWARDED

Respondeat Superior/Vicarious Liability 1. Recognition of respondeat superior/vicarious liability claim 2. Employer-employee relationship is required 2a. 2b. 3. 3a. 3b. 4. 5. Recognition of "control" or "right to control" Discussion of control issue (uniform, sign, policies) Recognition of frolic/detour issue Discussion of frolic/detour (only minor detour/returned to task)

1. 2. 2a. 2b. 3. 3a. 3b. 4. 5. 5a. 5b. 6. 7. 8.

Course and scope of employment required

PizzaFast would be liable for Dave's negligence. Definition of battery (harmful offensive touching or contact) 5a. 5b. Employer generally not liable for intentional torts. PizzaFast would be liable for Dave's intentional tort (battery)

Direct Liability 6. Elements of negligence (duty, breach, proximate cause, damages) 7. 8. Identification of negligence hiring/retention claim based on accidents Identification of negligence implementation/enforcement of thirty-minute policy.

6155164042

BLE Gradesheet v2.1

page 1 of 1

QUESTION 8 Plaintiff is a lifelong resident of Denver, Colorado. Last year, while driving in Denver, Plaintiff was struck and seriously injured by a car driven by Defendant. At the time of the accident Defendant was a citizen of England admitted to permanent residence and domiciled in Reno, Nevada. A police report identified Defendant as the sole cause of the accident. Defendant was charged and convicted of DUI in connection with the accident. Plaintiff commenced a civil action against Defendant in United States District Court for the District of Colorado. The complaint stated that the accident was the result of Defendants willful, wanton, reckless, and criminal misconduct. Plaintiff demanded compensatory damages for her personal injuries in the amount $60,000 and further demanded punitive damages in the amount of $50,000. The complaint alleged diversity of citizenship jurisdiction. Plaintiff personally delivered a copy of the summons and complaint to Defendant while he was eating at a restaurant in Reno, Nevada. One month after receiving the complaint, Defendant moved to Denver where he is now domiciled. Defendant filed a timely answer raising various defenses. He also served a counterclaim alleging that Plaintiff was partly responsible for the accident and demanding damages for the cost of repairing his car in the amount of $1000.

QUESTIONS: Discuss whether: 1. 2. 3. 4. 5. the U.S. District Court has subject matter jurisdiction. the court has personal jurisdiction. the defendant was properly served. venue is proper. the counterclaim is proper and within the courts jurisdiction.

7/08

DISCUSSION FOR QUESTION 8

I. Subject matter jurisdiction. Subject matter jurisdiction refers to a courts power to decide a particular kind of controversy. See Gene R. Shreve & Peter Raven-Hansen, Understanding Civil Procedure 107 (3d ed. 2002). Article III of the United States Constitution provides that the jurisdictional power of the United States extends to [c]ontroversies between citizens of different states . . . and between a State, or the citizens thereof, and foreign States, citizens, or subjects. U.S. Const. Art. III, 2. The Supreme Court has construed the diversity provision of the Judiciary Act of 1789 to require that no two parties on opposites sides of an action could be citizens of the same state. Thus, federal diversity jurisdiction is lacking if there are any litigants from the same state on opposing sides. See Prakash v. Am. Univ., 727 F.2d 1174, 1178, n.25 (D.C. Cir. 1984). A. Diversity Diversity of citizenship jurisdiction requires that there be complete diversity of citizenship between the plaintiffs on one side and defendants on the other and that the amount in controversy exceed $75,000 (exclusive of setoffs, interest, or costs). 28 U.S.C. 1332(a). In determining whether diversity of citizenship exists, the critical moment is the time at which the suit is commenced; under the Federal Rules, this refers to when the complaint is filed in the district court. Jack H. Friedenthal et al., Civil Procedure 27 (3d ed. 1999). See Saadeh v. Farouki, 107 F.3d 52, 57 (D.C. Cir. 1997); see generally Shreve and Raven-Hansen, supra at 122. There was diversity of citizenship between the plaintiff and defendant at the time the action is commenced. In general, to be a citizen of a state under the statute, a natural person must be both a citizen of the United States and be domiciled in a state. Wolfe v. Hartford Life & Annuity Ins. Co., 148 U.S. 389, 389 (1893)(holding averment of state residence insufficient); see generally Friedenthal et al., supra at 29; Shreve & Raven-Hansen, supra at 123. The test for state citizenship under the statute is domicile, and domicile is defined as one's "true, fixed, and permanent home. . .to which he has the intention of returning. . ." Stine v. Moore, 213 F.2d 446, 448 (5th Cir. 1954). Under these rules Plaintiff is a citizen of Colorado because she is a United States citizen domiciled in Colorado where she lives and works. An alien admitted to permanent residence "is deemed a citizen of the State in which such alien is domiciled." 28 U.S.C. 1332(a). Accordingly, Defendant's place of citizenship at the time the action was commenced was Nevada. His subsequent change of domicile and naturalization do not change the result. Saadeh, 107 F.3d at 57. Because the plaintiffs and defendant were citizens of different states at the time the action was commenced, there is diversity.

B. Amount in controversy The second part regarding subject matter jurisdiction relates to the amount in controversy. 28 U.S.C. 1332 (a) provides that district courts shall have original jurisdiction of all civil actions where the matter in controversy exceeds the sum or value of $75,000 exclusive of interest and costs. The amount-in-controversy requirement is normally governed by the monetary value of the relief claimed by the plaintiff in good faith. "[T]he sum claimed by the plaintiff controls if the claim is apparently made in good faith. It must appear to a legal certainty that the claim is really for less than the jurisdictional amount to justify a dismissal." St. Paul Mercury Indemnity Co. v. Red Cab Co., 303 U.S. 283, 288-89 (1938). Although the statute excludes interest and costs, a party may add together different kinds of damages, to satisfy the statutory amount. Punitive or exemplary damages can be added to the amount in controversy unless under applicable substantive law, a party is not legally entitled to such damages. See Friedenthal, supra at 45; Shreve & Raven-Hansen, supra at 126-27. Under Colorado law, a plaintiff may recover reasonable exemplary (punitive) damages, not exceeding the amount of actual damages recovered for willful and wanton misconduct. See Colo. Rev. Stat. 13-21-102(1)(a); White v. Hansen, 837 P.2d 1229, 1236 (Colo. 1992). Accordingly, the amount of damages demanded by Plaintiff is sufficient because the compensatory and punitive damages demanded in the complaint exceed $75,000.

II. Did the U.S. District Court have personal jurisdiction? The court would have personal jurisdiction if process was served properly. A federal court has territorial jurisdiction coextensive with the "jurisdiction of a court of general jurisdiction in the state in which the district court is located." Fed. R. Civ. P. 4(k)(1). The Colorado courts have specific personal jurisdiction over persons who enter the state and cause torts in the state for claims arising from their tortious conduct in the state. See Colo. Rev. Stat. 13-1-124(1)(b)(extending personal jurisdiction over causes of action against any person, resident or nonresident, who commits a tortious act within the state); Jenner & Block v. District Court, 590 P.2d 964, 965 (Colo. 1979)(extending long-arm statute to constitutional limits). Due process provides limits to the territorial reach of federal and state courts. U.S. Const. amends. V & XIV; International Shoe Co. v. Washington, 326 U.S. 310, 316 (1945); Pennoyer v. Neff, 95 U.S. (5 Otto) 714, 733 (1877). But long-arm jurisdiction over nonresident motorists for torts in the state does not violate due process. Hess v. Pawloski, 274 U.S. 352, 356-57 (1927), Zerr v. Norwood, 250 F. Supp. 1021, 1021 (D. Colo. 1966)(holding long-arm statute constitutionally applies to nonresident motorist who causes tort in Colorado).

III. Was there proper service of process? In federal court, process may be served pursuant to federal rule or pursuant to the methods authorized by the state law either of the place where the federal court is sitting or where

process is served. Fed. R. Civ. P. 4(e)&(h). See generally, Charles Alan Wright, Law of Federal Courts 64 at 445-46 (5th ed. 1994). Service of the summons and complaint was insufficient under both federal and Colorado state rules because Plaintiff personally delivered them, and the rules require service by a nonparty. Fed. R. Civ. P. 4(c)(2); Colo. R. Civ. P. 4(d). Accord Nev. R. Civ. P. 4(c). Accordingly, service of process is insufficient.

IV. Was venue proper? Venue is proper. At the time of the commencement of the action, the defendant was an alien admitted to permanent residence. An alien may be sued in any district. 28 U.S.C. 1391(d). Venue is also proper in an action based on diversity of citizenship in a judicial district in which a substantial part of the events or omissions giving rise to the claim occurred. 28 U.S.C. 1391(a)(2). Because the accident occurred in Colorado, the district of Colorado is proper. V. Was the counterclaim proper and within the courts jurisdiction? The counterclaim is proper. The Federal Rules permit certain counterclaims and require as a compulsory counterclaim a claim if it: (A) arises out of the transaction or occurrence that is the subject matter of the opposing partys claim; and (B) does not require adding another party over whom the court cannot acquire jurisdiction. Fed. R. Civ. P. 13(a). (Note: Rule amended 12/01/07) Because Defendants damages were the result of the same accident, they arose from the occurrence that is the subject matter of the plaintiffs claim, the counterclaim is authorized by federal rules. Moreover, a compulsory counterclaim is within the courts subject matter jurisdiction. Although the amount of damages demanded by the counterclaim does not independently satisfy the amount in controversy for diversity of citizenship jurisdiction, the federal court is authorized to exercise supplemental jurisdiction over a counterclaim in a case where the court has original jurisdiction over the plaintiffs claim, and the counterclaim arises from a common nucleus of operative facts. 28 U.S.C. 1367(a); United Mine Workers v. Gibbs, 383 U.S. 715, 725 (1966) (supplemental jurisdiction authorizes federal court to hear federal and state claims which share a common nucleus of operative fact such that all claims would ordinarily be expected to be tried in one judicial proceeding); Shreve & Raven-Hansen, supra at 147.

Conclusion The requirements for diversity of citizenship jurisdiction were met at the time the action was commenced, and the court was authorized to exercise jurisdiction over the person of the defendant provided he was served properly. Venue was also proper. Nevertheless, service of process was insufficient. If the action is dismissed, subject matter jurisdiction will be lacking when it is

recommenced, because Defendant will be deemed a citizen of the same state as Plaintiff as the time of commencement, and there will no longer be diversity of citizenship jurisdiction.

7/08

COLORADO SUPREME COURT Board of Law Examiners

JULY 2008 BAR EXAM Regrade

ESSAY Q8
ISSUE

SEAT
POINTS AWARDED

1. 2. 3. 4. 5. 6. 7. 8. 9. 10. 11. 12. 13. 14. 15. 16. 17.

Subject Matter jurisdiction if diversity of citizenship & amount exceeds $75,000. Must have diversity between all P's and D's on the other. Domicile = residence + intent to remain (both). Permanent resident alien is citizen of state of domicile. Later moved to CO doesn't destroy diversity. P from CO and D from NV therefore diversity. P can aggregate damages. Amount of damages sufficient because plaintiff is claiming in excess of $75,000. Minimum contacts necessary for jurisdiction. D drove vehicle in CO, so contacts sufficient. Personal service is acceptable means of service. Service not ok because P did it and a non-party is required Venue is proper where tort occurred. Accident occurred in CO, therefore, venue proper. Counterclaims required / compulsory if it arises out of the same transaction or occurrence (both). D's damages are from the same accident, therefore jurisdiction. D doesn't have to independently meet monetary requirement.

1. 2. 3. 4. 5. 6. 7. 8. 9. 10. 11. 12. 13. 14. 15. 16. 17.

6872165140

BLE Gradesheet v2.1

page 1 of 1

QUESTION 3 The Capital City Wolves are a professional football team. Dave Fan has season tickets to Wolves games. Very early in a Wolves game played after a snowfall, the head referee (Ref), announced a penalty against the Wolves. Fan became angry and, from his front-row seat, he threw a snowball at Ref. Ref saw the snowball coming and moved his head just in time to avoid being struck. Instead of striking Ref, the snowball struck the starting quarterback for the Wolves (Star). It hit the back of Stars helmeted head as he stood on the field talking with his teammates. Though Star was not injured by the impact, he was startled and jumped to the side in reaction to the snowball that struck him. In doing so, he collided with one of the Wolves coaches (Coach) who was on the field because play had stopped. Coach was not wearing any protective gear (obviously) and broke his arm when he was knocked to the ground by Star.

QUESTION: Discuss Fans potential liability, including damages, to Ref, Star, and Coach.

2/08

QUESTION 6 Before they got married over twenty years ago, Fred and Martha signed an agreement regarding the division of property and their financial responsibilities for their children should they divorce. They both were represented by separate attorneys during the negotiation process and each made full disclosure of their respective financial circumstances. The agreement provided that, in the event of divorce, they would bear equal financial responsibility for supporting any children of the marriage and that neither would be required to pay child support to the other. The agreement did not address maintenance. During the marriage, Fred and Martha had twin boys, William and Charles. In 2006, Martha started having an affair with Paul. She became pregnant in 2007. When Fred discovered Marthas infidelity, he filed for divorce. The divorce became final in 2007. Martha was not granted maintenance in the divorce. At that time, the twins were 18. William was stationed in Germany with the United States military and Charles was in college. Martha married Paul three months after the divorce became final, and the baby was born two weeks later. Shortly thereafter, Martha filed a motion requesting that Fred be required to pay child support for the baby. Fred responded by denying that the baby was his.

QUESTIONS: Discuss: 1. 2. whether the premarital agreement is enforceable in whole or in part; whether either party can be required to pay child support for, or otherwise financially support, William and Charles; and how the court should rule on the pending motion regarding child support for the baby.

3.

2/08

QUESTION 7 One day, the local First Federal Bank was robbed. Less than one mile from the bank, the police lawfully stopped Dan Defendant for speeding. Thinking he might be fleeing the bank robbery, the police took Defendant into custody and questioned him. Based on reports provided by bank tellers, Defendants proximity to the bank, and his speeding, Defendant was charged with bank robbery. The trial court appointed Al Attorney to represent Defendant. Attorney met with Defendant at the arraignment. Defendant explained that he was home with his mother at the time of the robbery, and that he was speeding because he was late for work. Attorney took notes, but never contacted Defendants mother or employer to attempt to verify Defendants story. Before trial, the prosecutor made a plea bargain offer to Attorney. Attorney rejected it outright, never communicating the offer to Defendant. At trial, the prosecutor presented the bank tellers as witnesses and they identified Defendant as the robber. The prosecution introduced a bank security camera video that showed a person resembling Defendant committing the robbery. After a brief deliberation, the jury found Defendant guilty. At the sentencing hearing, Defendant asserted that he was not guilty. He told the judge that he wanted to appeal. The judge appointed Carl Counselor to represent Defendant for purposes of the appeal. Counselor met with Defendant who explained that he wanted to appeal. Counselor told Defendant that he would take care of it. Counselor reviewed Attorneys notes from the trial and decided that there were not any meritorious issues he could raise on appeal. Counselor did not file a notice of appeal.

QUESTION: Discuss whether Defendants constitutional right to counsel was violated by the actions of his two attorneys.

2/08

Vous aimerez peut-être aussi